SlideShare a Scribd company logo
Professor DR Md . TOUFIQUR RAHMAN , FCPS, MD
Professor & Head, Cardiology, CMMC, Manikganj
drtoufiq19711@yahoo.com; drtoufiq1971@gmail.com
What are the causes of sinus bradycardia? (DU-04Ja)
Sinus bradycardia is a condition where the sinus node in the heart beats slower than the normal range
of 60-100 beats per minute. Some common causes of sinus bradycardia include:
 Vagal stimulation: This occurs due to an increased activity of the vagus nerve, which is
responsible for slowing down the heart rate.
 Medications: Certain medications like beta-blockers, calcium channel blockers, and digoxin can
cause sinus bradycardia.
 Hypothyroidism: Inadequate production of thyroid hormones can cause a decrease in metabolic
rate and lead to bradycardia.
 Increased intracranial pressure: High pressure within the skull due to conditions like head
injury, brain tumors or bleeding can affect the autonomic nervous system and cause bradycardia.
 Obstructive sleep apnea: Repeated episodes of apnea during sleep can cause bradycardia due to
decreased oxygen supply to the body.
 Aging: As the body ages, the electrical activity of the heart can slow down, leading to sinus
bradycardia.
Other causes of sinus bradycardia include viral infections, genetic disorders, and certain electrolyte
imbalances.
2. A 25 years old female presented with palpitation, on examination
her pulse was irregularly irregular. How will you assess and
investigate her? (DU- 05Ja)
The patient's presentation suggests the possibility of atrial fibrillation, which is a common arrhythmia
characterized by an irregularly irregular pulse. The following are the steps that can be taken to assess
and investigate the patient:
 History taking: Obtain a detailed history of the patient's symptoms, including the onset, duration,
and frequency of palpitations, associated symptoms, and any relevant medical history.
 Physical examination: Conduct a thorough physical examination, including a cardiovascular
examination, to assess the patient's heart sounds, rhythm, and rate. Check for any signs of heart
failure or underlying heart disease.
Professor DR Md . TOUFIQUR RAHMAN , FCPS, MD
Professor & Head, Cardiology, CMMC, Manikganj
drtoufiq19711@yahoo.com; drtoufiq1971@gmail.com
 Electrocardiogram (ECG): Perform an ECG to confirm the diagnosis of atrial fibrillation and to
determine the heart rate and rhythm. An ECG will also help rule out other arrhythmias or
underlying heart conditions.
 Blood tests: Check the patient's thyroid function, electrolyte levels, and other relevant blood tests
to identify any underlying conditions that may be causing the arrhythmia.
 Echocardiography: Perform an echocardiogram to assess the structure and function of the heart
and to identify any underlying heart disease.
 Holter monitor: Use a Holter monitor to monitor the patient's heart rate and rhythm over a 24-
hour period to identify any episodes of atrial fibrillation that may not be captured during a routine
ECG.
Other tests: Consider other tests, such as a stress test or electrophysiology study, if necessary, to
further evaluate the patient's heart function and arrhythmia.
In summary, the assessment and investigation of a patient with palpitations and an irregularly irregular
pulse would involve a comprehensive evaluation of the patient's symptoms, physical examination, ECG,
blood tests, echocardiogram, and possibly a Holter monitor or other tests to confirm the diagnosis and
identify any underlying conditions.
3. Write short note on : Stokes Adams syndrome. (DU- 06M)
Stokes-Adams syndrome, also known as complete heart block, is a condition characterized by episodes
of syncope (fainting) or seizures due to a sudden and temporary interruption of the heart's electrical
conduction system.
The underlying cause of Stokes-Adams syndrome is often attributed to degeneration or blockage of the
bundle of His, which is a crucial part of the heart's conduction system responsible for transmitting
electrical impulses from the atria to the ventricles. This can lead to a slower heart rate or complete
blockage of the electrical impulses to the ventricles.
Symptoms of Stokes-Adams syndrome can include dizziness, lightheadedness, loss of consciousness,
and seizures. Treatment may involve the use of a pacemaker to regulate the heart's electrical impulses
or medication to manage symptoms. In severe cases, surgical intervention may be required.
Professor DR Md . TOUFIQUR RAHMAN , FCPS, MD
Professor & Head, Cardiology, CMMC, Manikganj
drtoufiq19711@yahoo.com; drtoufiq1971@gmail.com
3. Write short note on : Stokes Adams syndrome. (DU- 06M)
Stokes-Adams syndrome is a medical condition characterized by episodes of fainting or syncope,
caused by a sudden drop in heart rate due to a disturbance in the heart's electrical conduction system.
Here are some key points about Stokes-Adams syndrome:
The syndrome is named after two physicians, William Stokes and Robert Adams, who first described
the condition in 1828 and 1827, respectively.
Stokes-Adams syndrome is caused by a blockage or interruption in the electrical signals that regulate
the heartbeat, leading to an abnormally slow heart rate (bradycardia) or even a complete pause in the
heart's rhythm (asystole).
The most common underlying cause of Stokes-Adams syndrome is a type of heart block known as
atrioventricular (AV) block, which occurs when the electrical signals that travel from the atria to the
ventricles are slowed or blocked.
Symptoms of Stokes-Adams syndrome can include dizziness, lightheadedness, fainting, confusion,
chest pain, and shortness of breath.
Treatment of Stokes-Adams syndrome depends on the severity of the symptoms and the underlying
cause of the heart block. Mild cases may be managed with medications such as atropine or
isoproterenol to increase heart rate, while more severe cases may require the implantation of a
pacemaker to regulate the heart's rhythm.
Complications of Stokes-Adams syndrome can include falls and injuries from fainting, as well as more
serious complications such as stroke or sudden cardiac arrest.
Angina Pectoris
1. What are the clinical menifestations of coronary heart disease ? (DU-09Ja)
Coronary heart disease (CHD) is a condition caused by the narrowing or blockage of the coronary
arteries, which supply blood and oxygen to the heart muscle. The clinical manifestations of CHD can
vary widely depending on the degree and location of coronary artery involvement. Some common
clinical manifestations of CHD include:
 Chest pain or discomfort: Angina pectoris is the most common symptom of CHD. The pain is
usually described as a tightness, pressure, squeezing, or burning sensation in the chest. It may also
be felt in the arms, neck, jaw, shoulder, or back.
 Shortness of breath: This symptom may occur with or without chest pain. It can be caused by
the heart not getting enough oxygen due to reduced blood flow.
Professor DR Md . TOUFIQUR RAHMAN , FCPS, MD
Professor & Head, Cardiology, CMMC, Manikganj
drtoufiq19711@yahoo.com; drtoufiq1971@gmail.com
 Palpitations: This is a sensation of rapid or irregular heartbeat. It may feel like the heart is
skipping a beat or beating too fast.
 Fatigue: Reduced blood flow to the heart can cause fatigue or weakness.
 Dizziness or lightheadedness: Reduced blood flow to the brain can cause dizziness or
lightheadedness.
 Syncope: Fainting may occur due to decreased blood flow to the brain.
Other symptoms: Nausea, vomiting, sweating, and anxiety may also occur.
It is important to note that some people with CHD may not experience any symptoms, especially in the
early stages of the disease.
2. Describe a typical angina. (DU- 07S)
A typical angina, also known as stable angina or exertional angina, is a type of chest pain that occurs
when the heart muscle does not get enough oxygenated blood due to narrowed coronary arteries. The
chest pain is usually described as a squeezing, pressure, heaviness, or tightness in the chest, which may
radiate to the left arm, neck, jaw, shoulder, or back. It is often triggered by physical exertion or
emotional stress and relieved by rest or sublingual nitroglycerin. The pain usually lasts for a few
minutes (up to 10 minutes) and is not associated with shortness of breath, nausea, or diaphoresis.
Patients with typical angina may have a history of atherosclerotic risk factors such as hypertension,
diabetes mellitus, hyperlipidemia, or smoking.
2. Describe a typical angina. (DU- 07S)
Typical angina is a clinical manifestation of coronary artery disease. It is characterized by chest pain or
discomfort that occurs with exertion, emotion, or other types of stress and is relieved by rest or
nitroglycerin. The following are the typical characteristics of angina:
 Location: The pain is typically felt behind the sternum or breastbone, but can also be felt in the
left arm, neck, jaw, or shoulders.
 Quality: The pain is described as a squeezing, pressure, heaviness, or tightness sensation.
 Duration: The pain usually lasts from a few seconds to a few minutes and is relieved by rest or
nitroglycerin.
 Triggers: The pain is typically triggered by physical exertion, emotional stress, exposure to cold
temperatures, or after a heavy meal.
Professor DR Md . TOUFIQUR RAHMAN , FCPS, MD
Professor & Head, Cardiology, CMMC, Manikganj
drtoufiq19711@yahoo.com; drtoufiq1971@gmail.com
 Response to nitroglycerin: The pain is relieved by nitroglycerin, a medication that dilates the
coronary arteries, allowing more blood flow to the heart muscle.
These characteristics are used to differentiate typical angina from other types of chest pain, such as
atypical angina, unstable angina, or myocardial infarction.
3. How will you clinical differentiate angina pectoris from esophageal pain?
(DU- 06Ja)
Differentiating angina pectoris from esophageal pain can be challenging because both can present with
similar symptoms such as retrosternal discomfort or burning pain. However, there are some clinical
differences that can help to distinguish between the two conditions:
 Onset and duration of pain: Angina pectoris typically presents with sudden onset of pain that
lasts for a few minutes (2-5 minutes), whereas esophageal pain often has a gradual onset and lasts
for a longer duration.
 Provoking factors: Angina pectoris is usually provoked by physical or emotional stress, whereas
esophageal pain is often provoked by ingestion of food or acid reflux.
 Radiation of pain: Angina pectoris commonly radiates to the left arm, shoulder, neck, jaw or
back, while esophageal pain typically radiates to the back, neck or arms.
 Response to nitroglycerin: Nitroglycerin can relieve the symptoms of angina pectoris, but has no
effect on esophageal pain.
 Associated symptoms: Angina pectoris may be associated with shortness of breath, sweating or
nausea, while esophageal pain may be associated with heartburn, regurgitation or difficulty
swallowing.
Therefore, taking a detailed history of the patient's symptoms, including the onset, duration, provoking
factors, radiation of pain, and associated symptoms, can help to differentiate angina pectoris from
esophageal pain. In addition, performing a physical examination and specific tests such as an
electrocardiogram (ECG) or an esophageal pH monitoring can also aid in the diagnosis.
How would you investigate a patient with stable angina with expected findings?
(DU-19Nov)
The investigation of a patient with stable angina typically involves the following:
Professor DR Md . TOUFIQUR RAHMAN , FCPS, MD
Professor & Head, Cardiology, CMMC, Manikganj
drtoufiq19711@yahoo.com; drtoufiq1971@gmail.com
 Electrocardiogram (ECG): A resting ECG may show evidence of previous myocardial
infarction (MI) or ischemia. During an angina episode, ECG changes may be present, such as ST-
segment depression or T-wave inversion.
 Exercise stress test: A stress test is a commonly used diagnostic test for stable angina. It involves
monitoring the ECG while the patient exercises, usually on a treadmill. The test helps to identify
ischemia-induced changes in the ECG and to determine the patient's exercise capacity.
 Echocardiography: This non-invasive imaging test uses sound waves to visualize the heart and
its function. It can help identify structural abnormalities, assess the function of the heart valves,
and estimate the ejection fraction (a measure of the heart's pumping efficiency).
 Coronary angiography: This is an invasive diagnostic test that involves injecting a contrast dye
into the coronary arteries and taking X-ray images to visualize any blockages or narrowing in the
arteries. It is usually reserved for patients with high-risk features or those who may require
invasive treatments such as angioplasty or bypass surgery.
 Blood tests: Cardiac biomarkers such as troponin may be measured to rule out acute coronary
syndrome as the cause of symptoms.
The expected findings of these investigations depend on the severity and extent of the coronary artery
disease. In stable angina, the ECG may show evidence of previous MI or ischemia, and the exercise
stress test may show evidence of ischemia-induced ECG changes. Echocardiography may show
reduced left ventricular function, and coronary angiography may reveal areas of narrowing or
blockages in the coronary arteries. Blood tests such as troponin are usually normal in stable angina.
In addition to investigating for stable angina, it is also important to investigate for risk factors that may
contribute to the development of coronary artery disease and unstable angina. Some of the
investigations that may be done include:
 Lipid profile: This measures the levels of cholesterol and other fats in the blood, which can
contribute to the development of atherosclerosis and subsequent angina.
 Fasting blood glucose: This test measures the amount of glucose in the blood after a period of
fasting. Elevated levels can indicate the presence of diabetes, which is a risk factor for coronary
artery disease and angina.
 Blood pressure: Elevated blood pressure is a risk factor for the development of coronary artery
disease and angina.
Professor DR Md . TOUFIQUR RAHMAN , FCPS, MD
Professor & Head, Cardiology, CMMC, Manikganj
drtoufiq19711@yahoo.com; drtoufiq1971@gmail.com
 ECG: This test can identify any abnormalities in the heart's electrical activity, which can indicate
the presence of coronary artery disease and angina.
 Stress test: This test involves monitoring the heart's response to physical activity or medication-
induced stress, and can help diagnose the presence of coronary artery disease and angina.
 Coronary angiography: This is an invasive test that involves injecting dye into the coronary
arteries to visualize any blockages or narrowing in the vessels. This test is typically reserved for
patients with suspected or confirmed unstable angina.
5. Write down the treatment of angina pectoris (DU-06Ja)
The treatment of angina pectoris may include:
 Lifestyle modifications: The patient may be advised to quit smoking, maintain a healthy weight,
exercise regularly, and follow a healthy diet.
 Medications: Medications such as nitrates, beta-blockers, calcium channel blockers, and
antiplatelet agents may be prescribed to control symptoms, reduce the risk of complications and
improve outcomes.
 Revascularization: Revascularization procedures such as angioplasty or coronary artery bypass
surgery may be recommended in some cases to improve blood flow to the heart and relieve
symptoms.
 Education and counseling: Patients with angina should be educated about their condition,
advised to recognize and avoid triggers, and counseled about stress reduction techniques.
 Follow-up care: Patients with angina require regular follow-up care to monitor their symptoms,
medications, and risk factors.
What do you mean by unstable angina? (DU- 07S) How would you manage a case
of unstable angina? (DU- 05Ja)
Unstable angina is a type of chest pain or discomfort that occurs when a person is at rest or
experiencing minimal exertion and is considered a medical emergency. It is caused by the disruption of
atherosclerotic plaques in the coronary arteries, leading to partial or complete blockage of blood flow
to the heart muscle.
Management of unstable angina involves immediate hospitalization and treatment to reduce the risk of
heart attack or cardiac arrest. The following steps can be taken:
Professor DR Md . TOUFIQUR RAHMAN , FCPS, MD
Professor & Head, Cardiology, CMMC, Manikganj
drtoufiq19711@yahoo.com; drtoufiq1971@gmail.com
 Oxygen therapy to increase the oxygen supply to the heart
 Nitroglycerin to relieve chest pain by dilating the blood vessels
 Aspirin or other antiplatelet agents to prevent the formation of blood clots
 Heparin or other anticoagulant medications to prevent the formation of blood clots
 Beta-blockers to reduce the workload on the heart and lower blood pressure
 Statins to lower cholesterol levels and reduce the risk of future cardiovascular events
 Invasive procedures such as angiography and percutaneous coronary intervention (PCI) or
coronary artery bypass grafting (CABG) may be necessary to treat underlying coronary artery
disease and prevent future episodes of unstable angina or myocardial infarction.
 The specific treatment plan will depend on the severity of the condition and individual patient
factors, and should be determined by a healthcare provider with expertise in managing unstable
angina.
Myocardial Infarction (MI)
*** 1. (a) How would you manage a case of acute myocardial infarction? (DU-
20M)
(a) Management of a case of acute myocardial infarction typically involves the following
steps:
 Rapid assessment and stabilization of the patient's condition: The patient's vital signs
and cardiac rhythm should be monitored, and interventions should be made to stabilize
them as needed. Oxygen should be administered to maintain oxygen saturation levels
above 94%. Nitroglycerin and morphine may be given to relieve pain and reduce anxiety.
 Early reperfusion therapy: The goal of reperfusion therapy is to restore blood flow to
the affected area of the heart as quickly as possible. This can be achieved through
percutaneous coronary intervention (PCI) or fibrinolytic therapy, depending on the
patient's clinical presentation and the availability of resources.
 Antiplatelet and anticoagulant therapy: Antiplatelet therapy with aspirin should be
initiated as soon as possible. Additional anticoagulant therapy with heparin or low-
molecular-weight heparin may also be given.
 Beta-blocker therapy: Beta-blockers should be started within the first 24 hours of an
acute myocardial infarction to reduce the risk of subsequent events.
Professor DR Md . TOUFIQUR RAHMAN , FCPS, MD
Professor & Head, Cardiology, CMMC, Manikganj
drtoufiq19711@yahoo.com; drtoufiq1971@gmail.com
 Statin therapy: Statins should be initiated in all patients with acute myocardial
infarction to reduce the risk of recurrent events.
 Cardiac rehabilitation: Cardiac rehabilitation should be started as soon as possible to
help the patient recover from the acute event and reduce the risk of future cardiac events.
(b) The management of a case of acute myocardial infarction should be tailored to the
individual patient's clinical presentation and underlying risk factors. The specific
interventions and therapies used will depend on the patient's hemodynamic stability, the
presence of complications, and the availability of resources. In general, the goal of treatment
is to restore blood flow to the affected area of the heart, minimize myocardial damage, and
prevent subsequent events. Close monitoring and prompt intervention are essential to
optimize outcomes.
(b) What are early complications of acute myocardial infarction? (DU-
M)
Some of the early complications of acute myocardial infarction (AMI) include:
Arrhythmias: It is a common complication of AMI and can occur due to damage to
the electrical conduction system of the heart. Arrhythmias can be life-threatening and
include ventricular tachycardia, ventricular fibrillation, and heart block.
Heart failure: AMI can damage a significant portion of the heart muscle, leading to
decreased cardiac function and heart failure.
Cardiogenic shock: In severe cases, AMI can cause cardiogenic shock, which occurs
when the heart is unable to pump enough blood to meet the body's demands.
Pericarditis: Inflammation of the pericardium, the sac surrounding the heart, can
occur after an AMI. This can cause chest pain and other symptoms.
Myocardial rupture: Rarely, the infarcted area of the heart muscle can rupture,
leading to severe internal bleeding and shock.
Professor DR Md . TOUFIQUR RAHMAN , FCPS, MD
Professor & Head, Cardiology, CMMC, Manikganj
drtoufiq19711@yahoo.com; drtoufiq1971@gmail.com
Ventricular septal defect: AMI can cause a hole to form in the wall that separates
the two lower chambers of the heart, leading to significant hemodynamic compromise.
Mitral regurgitation: Damage to the papillary muscles or the valve itself can lead to
mitral regurgitation, which can cause heart failure and other complications.
Thromboembolic events: Blood clots can form within the heart or in the blood
vessels leading to or from the heart, leading to stroke or other thromboembolic events.
Mention the principles of management of acute myocardial infarction.
(DU- 19M)
The principles of management of acute myocardial infarction include:
 Early recognition and diagnosis: Prompt recognition of MI symptoms and confirmation
through diagnostic tests such as electrocardiogram (ECG), cardiac biomarkers (troponin, CK-
MB), and imaging studies (echocardiography, coronary angiography).
 Reperfusion therapy: Restoration of blood flow to the ischemic myocardium through either
primary percutaneous coronary intervention (PCI) or fibrinolytic therapy. The choice of
therapy depends on the availability, time to treatment, and patient's clinical characteristics.
 Pharmacological therapy: Administration of antiplatelet agents, anticoagulants, beta-
blockers, angiotensin-converting enzyme inhibitors (ACEIs), angiotensin receptor blockers
(ARBs), and lipid-lowering agents such as statins.
 Monitoring and supportive care: Continuous monitoring of vital signs, ECG, and cardiac
biomarkers for any signs of worsening or complications such as arrhythmias, heart failure, or
cardiogenic shock. Oxygen therapy, pain relief, and hemodynamic support may also be
necessary.
 Risk factor modification and secondary prevention: Implementation of lifestyle
modifications such as smoking cessation, healthy diet, regular exercise, and stress reduction,
as well as initiation of medications for secondary prevention of cardiovascular events such as
aspirin, statins, and ACEIs/ARBs.
 Cardiac rehabilitation: A structured program of exercise training, risk factor modification,
and psychosocial support to improve functional capacity and quality of life after an acute MI.
Professor DR Md . TOUFIQUR RAHMAN , FCPS, MD
Professor & Head, Cardiology, CMMC, Manikganj
drtoufiq19711@yahoo.com; drtoufiq1971@gmail.com
Describe the characteristics of chest pain of acute myocardial
infarction. (DU-18Nov)
The characteristics of chest pain in acute myocardial infarction (AMI) are as follows:
 Location: The pain is typically felt in the center of the chest behind the sternum, although it
can also radiate to the left arm, neck, jaw, back, or epigastrium.
 Quality: The pain is usually described as a pressure, squeezing, tightness, or burning
sensation. It is often compared to a feeling of an elephant sitting on the chest or a heavy
weight.
 Severity: The pain is usually severe and intense, often described as the worst pain ever
experienced.
 Duration: The pain lasts for more than 20 minutes, but can last for several hours or longer.
 Associated symptoms: The pain is often associated with other symptoms such as sweating,
shortness of breath, nausea, vomiting, dizziness, and palpitations.
 Trigger factors: The pain may be triggered by physical or emotional stress, or may occur at
rest.
It is important to note that not all patients with AMI will experience chest pain, especially in the elderly,
women, and patients with diabetes. Therefore, other symptoms such as dyspnea, diaphoresis, nausea, or
vomiting may be the presenting features of AMI.
3. A 55 year old man has presented with central chest pain
precipitating with physical excretion & cold exposure, relieving with
taking rest for 3 months. On precordium auscultation, there is no
murmur. (DU-21M)
a. What is the most likely clinical diagnosis? Mention 2 (two) other features to differentiate
from musculoskeletal chest pain.
b. Write down the risk factors of the above condition.
Professor DR Md . TOUFIQUR RAHMAN , FCPS, MD
Professor & Head, Cardiology, CMMC, Manikganj
drtoufiq19711@yahoo.com; drtoufiq1971@gmail.com
a. The most likely clinical diagnosis is stable angina. Two features to differentiate from
musculoskeletal chest pain are:
The pain is typically precipitated by physical exertion or emotional stress and relieved by rest
or sublingual nitroglycerin.
The pain is described as a squeezing, pressure, heaviness, or tightness sensation in the chest,
often with radiation to the left arm, neck, jaw, or back.
b. The risk factors of stable angina include:
 Age (more common in older adults)
 Male gender
 Smoking
 High blood pressure
 High cholesterol levels
 Diabetes
 Family history of premature coronary artery disease
 Obesity or overweight
 Sedentary lifestyle
4. a. A 45 year old male presents with sudden severe chest pain
for 6 hours and ECG evidence is in favour of acute myocardial
infarction. How will you search for underlying risk factors?
In a patient with acute myocardial infarction, it is important to search for underlying risk factors that
may contribute to the development of the condition and affect the management plan. Some of the
common risk factors for myocardial infarction include:
 Age: Older age increases the risk of developing myocardial infarction.
 Family history of heart disease: Individuals with a family history of heart disease are at a
higher risk of developing myocardial infarction.
 Smoking: Smoking is a major risk factor for myocardial infarction.
 High blood pressure: Individuals with high blood pressure are at an increased risk of
myocardial infarction.
 High cholesterol: High levels of cholesterol in the blood can increase the risk of myocardial
infarction.
 Diabetes: Individuals with diabetes are at a higher risk of developing myocardial infarction.
 Obesity: Being overweight or obese increases the risk of myocardial infarction.
Professor DR Md . TOUFIQUR RAHMAN , FCPS, MD
Professor & Head, Cardiology, CMMC, Manikganj
drtoufiq19711@yahoo.com; drtoufiq1971@gmail.com
 Sedentary lifestyle: Lack of physical activity can increase the risk of myocardial infarction.
 Stress: Chronic stress can increase the risk of developing myocardial infarction.
To search for underlying risk factors in a patient with acute myocardial infarction, a detailed history
and physical examination should be performed. This may include asking about family history, smoking
history, dietary habits, exercise habits, and medical history (such as diabetes or hypertension). Blood
tests, such as lipid profile and glucose level, can also be performed to assess for underlying risk factors.
b. Mention the clinical conditions related to excess environment heat.
(DU-17Ja)
Clinical conditions related to excess environmental heat include:
 Heat cramps: painful muscle spasms that occur during or after intense exercise in hot
environments
 Heat exhaustion: a condition characterized by heavy sweating, weakness, dizziness, nausea,
and headache due to dehydration and electrolyte imbalance
 Heat syncope: fainting or dizziness caused by a sudden drop in blood pressure due to
standing for prolonged periods in a hot environment
 Heat rash: small red bumps or blisters on the skin due to sweating and clogged sweat ducts
 Heatstroke: a life-threatening condition characterized by a body temperature of 104°F or
higher, confusion, seizures, and loss of consciousness. It is considered a medical emergency
and requires immediate treatment.
5. A 60 year old smoker is admitted with acute central chest pain for 2 hours. His
ECG shows ST elevation. (DU-16Ja)
a. What immediate management would you take for the patient?
The immediate management for the patient with acute central chest pain and ST elevation on
ECG would be to initiate reperfusion therapy as soon as possible. This can be done with either
fibrinolytic therapy or primary percutaneous coronary intervention (PCI) depending on the
available resources and the time since the onset of symptoms.
Other immediate management measures would include administering oxygen to maintain
oxygen saturation above 94%, initiating antiplatelet therapy with aspirin and P2Y12 inhibitors,
and providing pain relief with intravenous morphine. The patient should also be closely
monitored for complications such as ventricular arrhythmias and cardiogenic shock
Professor DR Md . TOUFIQUR RAHMAN , FCPS, MD
Professor & Head, Cardiology, CMMC, Manikganj
drtoufiq19711@yahoo.com; drtoufiq1971@gmail.com
 Call for emergency medical services to transfer the patient to the nearest hospital
with a cardiac catheterization laboratory and 24-hour primary percutaneous coronary
intervention (PCI) capabilities.
 Administer aspirin chewable tablet of 300 mg or 4 tablets of 75 mg each orally, and
nitroglycerin sublingual tablet or spray every 5 minutes up to three doses as needed to
relieve chest pain.
 Initiate oxygen therapy with a target saturation level of at least 94%.
 Administer a loading dose of a P2Y12 inhibitor (e.g., clopidogrel 600 mg,
prasugrel 60 mg, or ticagrelor 180 mg) orally or through a nasogastric tube.
 Perform an electrocardiogram (ECG) and repeat it every 10-15 minutes if there is
persistent chest pain.
 Obtain venous access and start an intravenous infusion of heparin or bivalirudin, a
direct thrombin inhibitor.
 Monitor blood pressure, heart rate, oxygen saturation, and ECG changes
continuously.
 Consider providing analgesia with opioids if chest pain persists despite the
administration of nitroglycerin.
 Reevaluate the patient and consider the need for urgent revascularization by primary
PCI or thrombolytic therapy in consultation with a cardiologist.
It is crucial to remember that the management of acute myocardial infarction is time-sensitive,
and early recognition and prompt treatment can save lives and prevent complications.
b. The patient develops dyspnoea on the following day. What additional
measures will you take to manage the patient?
If the patient develops dyspnea following an acute myocardial infarction, it may suggest the
development of heart failure or a complication such as a ventricular septal defect. The
following additional measures may be taken to manage the patient:
 Administer supplemental oxygen to improve oxygen saturation levels and relieve
dyspnea.
 Administer diuretics to reduce fluid overload and relieve dyspnea associated with
heart failure.
 Administer nitrates to dilate blood vessels and reduce cardiac workload, thereby
relieving dyspnea.
 Monitor vital signs, fluid balance, and urine output to assess the patient's response to
treatment and detect any signs of worsening heart failure.
Professor DR Md . TOUFIQUR RAHMAN , FCPS, MD
Professor & Head, Cardiology, CMMC, Manikganj
drtoufiq19711@yahoo.com; drtoufiq1971@gmail.com
 Consider further investigations such as echocardiography to assess the patient's left
ventricular function and identify any complications such as ventricular septal defect.
 Consult with a cardiologist for further management and possible interventions, such
as cardiac catheterization or coronary artery bypass graft surgery, depending on the
underlying cause of the dyspnea.
6. a. What is acute coronary syndrome? How do confirm acute
myocardial infraction? (DU-15Ju)
Acute coronary syndrome (ACS) refers to a group of clinical conditions that are caused by a
sudden reduction or occlusion of blood flow to the heart due to coronary artery disease. The
three clinical conditions that make up ACS are unstable angina, non-ST segment elevation
myocardial infarction (NSTEMI), and ST-segment elevation myocardial infarction (STEMI).
To confirm acute myocardial infarction (AMI), several diagnostic tests can be used, including:
 Electrocardiogram (ECG): An ECG measures the electrical activity of the heart and
can detect changes in the heart's rhythm and electrical conduction. In AMI, the ECG
may show ST-segment elevation, indicating myocardial injury.
 Cardiac biomarkers: These are blood tests that measure the levels of enzymes and
proteins released by damaged heart muscle cells. Elevated levels of cardiac
biomarkers such as troponin indicate myocardial damage and help to confirm the
diagnosis of AMI.
 Imaging studies: Imaging studies such as echocardiography or cardiac MRI may be
used to assess the extent and severity of myocardial damage and to help guide
treatment decisions.
b. What is the difference between NSTEMI and STEMI? (DU-15Ju)
The main difference between NSTEMI and STEMI is the extent and severity of myocardial
damage. In NSTEMI, the blood flow to the heart is partially occluded, and there is partial
myocardial damage. In contrast, in STEMI, there is complete occlusion of the coronary artery,
leading to complete myocardial damage.
Professor DR Md . TOUFIQUR RAHMAN , FCPS, MD
Professor & Head, Cardiology, CMMC, Manikganj
drtoufiq19711@yahoo.com; drtoufiq1971@gmail.com
Other differences between NSTEMI and STEMI include:
 ECG changes: In NSTEMI, the ECG may show ST-segment depression or T-wave
inversion, indicating myocardial ischemia. In STEMI, the ECG typically shows ST-
segment elevation, indicating myocardial injury.
 Cardiac biomarkers: In both NSTEMI and STEMI, cardiac biomarkers such as
troponin are elevated, but the levels are typically higher in STEMI due to the more
extensive myocardial damage.
 Treatment: The treatment of NSTEMI and STEMI differs, with STEMI requiring
more urgent and aggressive intervention, such as reperfusion therapy, to restore blood
flow to the affected area of the heart. NSTEMI is usually managed with medical
therapy and may require less urgent intervention.
b. Mention immediate management of this condition. Mention the
principles of management of acute coronary syndrome (DU-18Ju)
Immediate management of acute coronary syndrome includes:
 Administering aspirin: Aspirin is a blood thinner and helps in preventing the formation of
blood clots. It should be chewed or crushed and swallowed immediately after a heart attack is
suspected.
 Administering nitroglycerin: Nitroglycerin helps in relaxing the blood vessels and easing the
workload on the heart. It should be given sublingually (under the tongue) or intravenously.
 Administering morphine: Morphine helps in relieving the pain associated with acute
coronary syndrome. It also reduces the workload on the heart.
 Supplemental oxygen: Oxygen should be given to the patient to ensure adequate oxygen
supply to the heart.
 Beta-blockers: Beta-blockers help in reducing the workload on the heart and are given orally
or intravenously.
The principles of management of acute coronary syndrome include:
Professor DR Md . TOUFIQUR RAHMAN , FCPS, MD
Professor & Head, Cardiology, CMMC, Manikganj
drtoufiq19711@yahoo.com; drtoufiq1971@gmail.com
 Early recognition: Acute coronary syndrome should be recognized as early as possible, and
treatment should be started immediately.
 Prompt revascularization: Revascularization, either through angioplasty or bypass surgery,
should be done as soon as possible to restore blood flow to the affected area of the heart.
 Pharmacological therapy: Medications such as aspirin, nitroglycerin, beta-blockers, and
statins should be given to manage the symptoms and reduce the risk of future cardiovascular
events.
 Risk factor management: Risk factors such as hypertension, diabetes, and high cholesterol
should be managed to prevent future cardiovascular events.
 Cardiac rehabilitation: Cardiac rehabilitation, including physical exercise, lifestyle
modifications, and stress management, should be initiated to improve the patient's overall
cardiovascular health and prevent future events.
7. Name the enzymes those may be elevated in acute MI. (DU-09Ja)
In the context of acute MI, the following enzymes may be elevated:
 Creatine kinase (CK-MB): CK-MB levels rise 3-6 hours after MI onset and peak at 12-24
hours. It returns to normal within 2-3 days.
 Troponin I or T: Troponin levels rise 3-6 hours after MI onset and remain elevated for up to
2 weeks.
 Myoglobin: Myoglobin levels rise within 2-3 hours after MI onset and peak within 6-9 hours.
It returns to normal within 24 hours.
Elevation of these enzymes in the appropriate clinical context can help confirm the diagnosis of acute
MI.
8. Write down the management of acute MI (DU-10Ju)
Management of Acute Myocardial Infarction (MI) involves the following steps:
 Initial assessment and stabilization:
This includes monitoring vital signs, administering oxygen therapy, establishing IV access, providing
pain relief with medications such as nitroglycerin and morphine, and assessing the need for urgent
reperfusion therapy.
 Reperfusion therapy:
Professor DR Md . TOUFIQUR RAHMAN , FCPS, MD
Professor & Head, Cardiology, CMMC, Manikganj
drtoufiq19711@yahoo.com; drtoufiq1971@gmail.com
The aim of reperfusion therapy is to restore blood flow to the blocked coronary artery as soon as
possible. It can be achieved either by primary percutaneous coronary intervention (PCI) or
thrombolytic therapy. Primary PCI is preferred if it can be performed within 90 minutes of first medical
contact.
 Antiplatelet and anticoagulant therapy:
Aspirin is given immediately and continued indefinitely. Other antiplatelet agents such as clopidogrel
or ticagrelor are also given. Anticoagulant therapy with heparin or enoxaparin is started and continued
for several days.
 Beta-blocker therapy:
Beta-blockers are given to reduce the risk of recurrent MI, arrhythmias, and heart failure. They should
be started as soon as possible and continued indefinitely.
 ACE inhibitors/ARBs:
These drugs are given to reduce the risk of recurrent MI, heart failure, and death. They should be
started within the first 24 hours of MI and continued indefinitely.
 Statin therapy:
Statins are given to reduce LDL cholesterol levels and decrease the risk of recurrent MI and death.
They should be started as soon as possible and continued indefinitely.
 Cardiac rehabilitation:
This involves exercise training, lifestyle modification, and risk factor reduction. It should be started as
soon as possible after discharge and continued for several weeks to months.
 Complication management:
Management of complications such as heart failure, cardiogenic shock, arrhythmias, and mechanical
complications such as papillary muscle rupture or ventricular septal defect should be done promptly.
 Long-term follow-up:
Patients with acute MI should be followed up regularly for several years to monitor for recurrent events
and optimize secondary prevention measures.
9. A 45 year old male presents with sudden severe chest pain for 1 hour.
(DU- 19Ja)
a. Make a check list history and physical signs to find out the causes.
b. How would you manage this case?
a. Check list history and physical signs to find out the causes of sudden severe chest pain in a 45-year-
old male:
History:
 Time of onset of pain
 Duration and frequency of pain
Professor DR Md . TOUFIQUR RAHMAN , FCPS, MD
Professor & Head, Cardiology, CMMC, Manikganj
drtoufiq19711@yahoo.com; drtoufiq1971@gmail.com
 Quality and intensity of pain
 Radiation of pain
 Any associated symptoms like sweating, nausea, vomiting, breathlessness, palpitation,
dizziness, or syncope
 Any past history of similar episodes
 Any past medical history of hypertension, diabetes mellitus, dyslipidemia, smoking, or family
history of coronary artery disease
Physical examination:
 Vital signs: blood pressure, heart rate, respiratory rate, and oxygen saturation
 Cardiac examination: rhythm, rate, murmurs, and gallops
 Respiratory examination: breath sounds, crepitations, or wheezing
 Abdominal examination: tenderness, guarding, or distension
 Neurological examination: level of consciousness, pupils, and motor power
b. Management of a case with sudden severe chest pain for 1 hour:
 Assess the ABCs (airway, breathing, and circulation)
 Administer oxygen via a nasal cannula at a flow rate of 4-6 L/min
 Administer aspirin 300mg orally or 150-300mg chewed and swallowed as soon as possible
 Administer sublingual glyceryl trinitrate (GTN) 0.4 mg every 5 minutes up to a maximum of 3
doses, if there is no contraindication
 Obtain a 12-lead electrocardiogram (ECG) as soon as possible
 Consider early reperfusion therapy if indicated (thrombolytic therapy or primary percutaneous
coronary intervention) within 12 hours of the onset of symptoms
 Administer morphine sulfate 2-4mg intravenously, if there is no contraindication and the pain
persists after treatment with GTN
 Monitor the patient's vital signs and cardiac rhythm closely
 Admit the patient to the coronary care unit for further management and observation
10. A 60 year old man has been admitted with severe central chest pain. His ECG
shows ST elevation. (DU-14Ju)
a. How do you manage the patient in a Medical College Hospital?
b. The patient developed dyspnoea on the following day. What additional measures will
you take to manage the patient?
a. The management of the patient with severe central chest pain and ST elevation on ECG in a Medical
College Hospital typically involves the following steps:
Professor DR Md . TOUFIQUR RAHMAN , FCPS, MD
Professor & Head, Cardiology, CMMC, Manikganj
drtoufiq19711@yahoo.com; drtoufiq1971@gmail.com
 Rapid assessment: The patient should be assessed quickly to determine the severity of their
condition and the need for urgent intervention.
 Oxygen therapy: Oxygen should be administered immediately to improve oxygenation.
 Pain relief: Analgesics such as morphine sulfate or fentanyl should be given for pain relief.
 Antiplatelet therapy: Aspirin should be given as soon as possible to reduce the risk of further
cardiac events.
 Reperfusion therapy: If available, reperfusion therapy (such as thrombolysis or primary
percutaneous coronary intervention) should be initiated within the recommended time frame
to restore blood flow to the affected area of the heart.
 Monitoring: The patient should be monitored closely for any changes in their condition,
including vital signs, ECG, and cardiac enzymes.
 Risk factor management: The patient should receive counseling and interventions to manage
their risk factors for cardiovascular disease, such as smoking cessation, blood pressure control,
and lipid management.
b. If the patient develops dyspnea on the following day after an acute myocardial infarction, additional
measures may be needed to manage their condition, including:
 Oxygen therapy: Supplemental oxygen should be given to maintain oxygen saturation levels.
 Diuretics: Diuretics may be used to reduce fluid overload and relieve pulmonary congestion.
 Nitrates: Nitrates such as nitroglycerin may be given to reduce the workload of the heart and
improve symptoms.
 ACE inhibitors: ACE inhibitors may be started to reduce the risk of further cardiac events
and improve outcomes.
 Monitoring: The patient should be monitored closely for any signs of worsening heart failure,
including changes in vital signs, ECG, and cardiac enzymes.
11. A 45 years old male smoker presented with chest pain. (DU-13Ju, 10Ju)
a. Mention the important characteristics of pain which will suggest MI.
Professor DR Md . TOUFIQUR RAHMAN , FCPS, MD
Professor & Head, Cardiology, CMMC, Manikganj
drtoufiq19711@yahoo.com; drtoufiq1971@gmail.com
b. Give an outline of management of acute myocardial infraction.
a. The important characteristics of chest pain suggestive of myocardial infarction are:
 Sudden onset of pain
 Chest pain that is severe and crushing in nature
 Pain that radiates to the left arm, neck, jaw, back, or shoulder
 Pain that is not relieved by rest or nitroglycerin
 Pain that lasts for more than 20 minutes
b. The management of acute myocardial infarction includes the following steps:
 Aspirin: Administer 325mg of aspirin as soon as possible.
 Oxygen: Administer oxygen if the patient is hypoxic.
 Nitroglycerin: Administer sublingual nitroglycerin for chest pain.
 Analgesics: Administer pain relief, such as morphine, if the patient has persistent chest pain.
 Reperfusion: Reperfusion therapy should be given within 12 hours of symptom onset. This
includes either thrombolytic therapy or percutaneous coronary intervention (PCI).
 Anticoagulants: Administer anticoagulant therapy, such as heparin or low molecular weight
heparin.
 Antiplatelet therapy: Administer antiplatelet therapy, such as clopidogrel or ticagrelor.
 Beta blockers: Administer beta blockers in the absence of contraindications.
 ACE inhibitors or ARBs: Administer angiotensin-converting enzyme (ACE) inhibitors or
angiotensin receptor blockers (ARBs) in the absence of contraindications.
 Statins: Administer statins to all patients with acute myocardial infarction.
In addition to the above steps, it is important to monitor the patient for complications and provide
appropriate management. Close monitoring of vital signs, cardiac rhythm, and oxygen saturation is
necessary. Patients with cardiogenic shock may require mechanical support, such as an intra-aortic
balloon pump. Patients with arrhythmias may require antiarrhythmic therapy or electrical cardioversion.
12. A 50 years old smoker presents with acute central chest pain. (DU-12Ju)
a. What historical features and physical sign will suggest acute coronary syndrome?
b. Mention the immediate management measures in this case.
a. Historical features and physical signs that may suggest acute coronary syndrome in a 50-year-
old smoker with acute central chest pain include:
 Sudden onset of pain
 Crushing or squeezing pain, often described as "heaviness" or "tightness"
 Pain may radiate to the arms (usually left), neck, jaw, back or upper abdomen
 Shortness of breath
Professor DR Md . TOUFIQUR RAHMAN , FCPS, MD
Professor & Head, Cardiology, CMMC, Manikganj
drtoufiq19711@yahoo.com; drtoufiq1971@gmail.com
 Nausea or vomiting
 Sweating
 Pallor or grayish color of the skin
 Irregular heartbeat
 Decreased blood pressure
Physical examination may reveal:
 Rapid or irregular heartbeat
 Low blood pressure
 Crackles in the lungs (suggesting heart failure)
 Murmurs (suggesting valvular heart disease)
b. The immediate management measures for a patient with acute coronary syndrome include:
 Oxygen therapy: to maintain oxygen saturation above 94%
 Aspirin: to prevent further clotting
 Nitroglycerin: to relieve chest pain and improve blood flow
 Morphine: if nitroglycerin fails to relieve pain or if the patient is anxious
 Beta blockers: to reduce heart rate and blood pressure and prevent further damage to the heart
muscle
 Anticoagulants: to prevent further clotting and reduce the risk of stroke
 Reperfusion therapy: either fibrinolytic therapy or percutaneous coronary intervention (PCI)
to restore blood flow to the heart muscle.
After the immediate management measures, the patient will need further evaluation, risk stratification
and long-term management to prevent further cardiac events.
* 13. Give complications of MI. (DU-11Ja, 10Ja)
Complications of MI (Myocardial Infarction) can include:
 Arrhythmias: Abnormal heart rhythms due to the damage to the heart muscle can lead to
various types of arrhythmias, including ventricular tachycardia or fibrillation, which can be
life-threatening.
 Heart failure: MI can lead to heart failure, which occurs when the heart can't pump enough
blood to meet the body's needs. This can lead to symptoms such as shortness of breath, fatigue,
and swelling in the legs and ankles.
 Cardiogenic shock: This is a severe form of heart failure where the heart is unable to pump
enough blood to the vital organs, leading to organ failure and potentially death.
 Pericarditis: Inflammation of the lining surrounding the heart can occur following an MI.
Professor DR Md . TOUFIQUR RAHMAN , FCPS, MD
Professor & Head, Cardiology, CMMC, Manikganj
drtoufiq19711@yahoo.com; drtoufiq1971@gmail.com
 Thrombosis and embolism: Blood clots can form in the damaged heart muscle and may
break off and travel to other parts of the body, causing blockages.
 Ventricular aneurysm: MI can lead to a bulge in the weakened heart muscle, which can
cause arrhythmias, heart failure, and further MI.
 Dressler's syndrome: This is a rare autoimmune response to an MI, which can cause fever,
chest pain, and pericarditis.
 Stroke: Blood clots formed during MI can break off and travel to the brain, causing a stroke.
 Post-infarction angina: Chest pain can persist even after the MI has resolved.
 Sudden cardiac death: This can occur due to arrhythmias or other complications of MI.
1. A 30 years old man recently detected as having hypertension. (DU-
20m)
(a) Make a check list of clinical history to find out the causes of hypertension.
(b) Write down the investigation plan for him.
(a) Check list of clinical history to find out the causes of hypertension:
 Family history of hypertension
 History of previous kidney disease or renal abnormalities
 Sleep apnea or other respiratory issues
 Hormonal disorders such as thyroid disease or Cushing's syndrome
 Alcohol consumption
 Smoking history
 Drug history, including over-the-counter medications and illegal substances
 Dietary habits and physical activity levels
 Stressful life events or chronic stress
(b) Investigation plan for a patient with hypertension may include:
 Blood tests: To check for any underlying health conditions, such as diabetes, kidney disease,
or high cholesterol.
 Urine tests: To check for any kidney abnormalities or protein in the urine.
 ECG (Electrocardiogram): To check for any abnormalities in heart rhythm or structure.
Professor DR Md . TOUFIQUR RAHMAN , FCPS, MD
Professor & Head, Cardiology, CMMC, Manikganj
drtoufiq19711@yahoo.com; drtoufiq1971@gmail.com
 Ambulatory blood pressure monitoring (ABPM): A 24-hour blood pressure measurement
to assess blood pressure variability and determine if the patient has white-coat hypertension or
masked hypertension.
 Echocardiogram: To evaluate the structure and function of the heart and to rule out any
underlying heart disease.
 Imaging tests: Such as CT scan, MRI, or angiography to evaluate blood vessels and to check
for any abnormalities that may be contributing to hypertension.
 Stress test: To evaluate the patient's heart function during exercise and to determine the
presence of any underlying heart disease.
 Genetic testing: In some cases, genetic testing may be performed to determine if the patient
has a genetic predisposition to hypertension or underlying heart disease.
The investigation plan may vary based on the individual patient's history, symptoms, and physical
examination findings.
2. A 35 year old male recently detected as hypertension for 3 months. (DU-19Ja)
a. Make a check list and physical sign to find out the causes of hypertension.
b. Write down the investigation plan for him.
a. The checklist and physical signs to find out the causes of hypertension in a 35-year-old male
are:
Checklist:
 Family history of hypertension
 History of kidney disease
 History of diabetes mellitus
 Lifestyle factors: diet, exercise, tobacco and alcohol use
 Medication history
 Endocrine disorders such as Cushing's syndrome or hyperthyroidism
 Sleep apnea
Physical signs:
 Elevated blood pressure reading on at least two occasions
 Evidence of end-organ damage such as retinal changes, proteinuria, or left ventricular
hypertrophy
 Obesity
 Thyroid enlargement or nodules
 Abdominal bruit
 Palpable renal masses
Professor DR Md . TOUFIQUR RAHMAN , FCPS, MD
Professor & Head, Cardiology, CMMC, Manikganj
drtoufiq19711@yahoo.com; drtoufiq1971@gmail.com
b. The investigation plan for a 35-year-old male with hypertension may include:
 Complete blood count (CBC)
 Fasting blood glucose level
 Lipid profile
 Renal function tests including serum creatinine, blood urea nitrogen (BUN), and estimated
glomerular filtration rate (eGFR)
 Urinalysis
 Electrocardiogram (ECG)
 Echocardiogram
 Chest X-ray
 Thyroid function tests
 Sleep study (if sleep apnea is suspected)
 24-hour urine collection for catecholamines and metanephrines (if pheochromocytoma is
suspected)
Additional tests may be ordered based on the initial findings and clinical suspicion. The investigation
plan should also include regular monitoring of blood pressure and close follow-up with a healthcare
provider to assess response to treatment and adjust management accordingly.

More Related Content

Similar to MBBS QUESTION ANSWER 3 CARD.pdf

Satish ppt disorders of heart and lungs
Satish  ppt  disorders of heart and lungsSatish  ppt  disorders of heart and lungs
Satish ppt disorders of heart and lungs
SSãì SSæthìßh
 
I s c h a e m i a
I s c h a e m i aI s c h a e m i a
I s c h a e m i a
Sayed Sileem
 
diagnosis and management of ischemic cerebrovascular disease by Ismail Surchi
diagnosis and management of ischemic cerebrovascular disease by Ismail Surchidiagnosis and management of ischemic cerebrovascular disease by Ismail Surchi
diagnosis and management of ischemic cerebrovascular disease by Ismail Surchi
Ismail Surchi
 
Pathophysiology of angina and atheroslerosis
Pathophysiology of angina and atheroslerosisPathophysiology of angina and atheroslerosis
Pathophysiology of angina and atheroslerosis
Koppala RVS Chaitanya
 
Lesson plan on myocardial infarction
Lesson plan on myocardial infarctionLesson plan on myocardial infarction
Lesson plan on myocardial infarction
kalyan kumar
 
Cardiovascular diseases modified
Cardiovascular diseases modifiedCardiovascular diseases modified
Cardiovascular diseases modifiedxtrm nurse
 
9. coronary heart disease
9. coronary heart disease9. coronary heart disease
9. coronary heart disease
Ahmad Hamadi
 
13942698
1394269813942698
13942698
sumairariaz5
 
Periodontal Treatment of Medically Compromised Patients [Autosaved].pptx
Periodontal Treatment of Medically Compromised Patients [Autosaved].pptxPeriodontal Treatment of Medically Compromised Patients [Autosaved].pptx
Periodontal Treatment of Medically Compromised Patients [Autosaved].pptx
ANIL KUMAR
 
Heart block
Heart block Heart block
Heart block
ROMAN BAJRANG
 
3. coronary artery disease (cad)
3. coronary artery disease (cad)3. coronary artery disease (cad)
3. coronary artery disease (cad)
khizraarshed
 
Coronary insufficiency
Coronary insufficiencyCoronary insufficiency
Coronary insufficiency
Devil's Drill
 
Stroke
StrokeStroke
Stroke
Natiatme
 
CARDIAC EMERGENCY
CARDIAC EMERGENCYCARDIAC EMERGENCY
CARDIAC EMERGENCY
Shalu Udhay
 
Ischemic heart disease (IHD) pathophysiology , types , its causes and treatment
Ischemic heart disease (IHD) pathophysiology , types , its causes and treatmentIschemic heart disease (IHD) pathophysiology , types , its causes and treatment
Ischemic heart disease (IHD) pathophysiology , types , its causes and treatment
Home
 
Cerebrovascular Accident
Cerebrovascular AccidentCerebrovascular Accident
Cerebrovascular Accident
Maria Guia Nelson
 

Similar to MBBS QUESTION ANSWER 3 CARD.pdf (20)

Satish ppt disorders of heart and lungs
Satish  ppt  disorders of heart and lungsSatish  ppt  disorders of heart and lungs
Satish ppt disorders of heart and lungs
 
I s c h a e m i a
I s c h a e m i aI s c h a e m i a
I s c h a e m i a
 
diagnosis and management of ischemic cerebrovascular disease by Ismail Surchi
diagnosis and management of ischemic cerebrovascular disease by Ismail Surchidiagnosis and management of ischemic cerebrovascular disease by Ismail Surchi
diagnosis and management of ischemic cerebrovascular disease by Ismail Surchi
 
Pathophysiology of angina and atheroslerosis
Pathophysiology of angina and atheroslerosisPathophysiology of angina and atheroslerosis
Pathophysiology of angina and atheroslerosis
 
Lesson plan on myocardial infarction
Lesson plan on myocardial infarctionLesson plan on myocardial infarction
Lesson plan on myocardial infarction
 
Cardiovascular diseases modified
Cardiovascular diseases modifiedCardiovascular diseases modified
Cardiovascular diseases modified
 
9. coronary heart disease
9. coronary heart disease9. coronary heart disease
9. coronary heart disease
 
13942698
1394269813942698
13942698
 
Periodontal Treatment of Medically Compromised Patients [Autosaved].pptx
Periodontal Treatment of Medically Compromised Patients [Autosaved].pptxPeriodontal Treatment of Medically Compromised Patients [Autosaved].pptx
Periodontal Treatment of Medically Compromised Patients [Autosaved].pptx
 
Heart block
Heart block Heart block
Heart block
 
Cardiovascular emergencies 2011
Cardiovascular emergencies 2011Cardiovascular emergencies 2011
Cardiovascular emergencies 2011
 
3. coronary artery disease (cad)
3. coronary artery disease (cad)3. coronary artery disease (cad)
3. coronary artery disease (cad)
 
Coronary insufficiency
Coronary insufficiencyCoronary insufficiency
Coronary insufficiency
 
Stroke
StrokeStroke
Stroke
 
Angina pectoris
Angina pectorisAngina pectoris
Angina pectoris
 
CARDIAC EMERGENCY
CARDIAC EMERGENCYCARDIAC EMERGENCY
CARDIAC EMERGENCY
 
Ischemic heart disease (IHD) pathophysiology , types , its causes and treatment
Ischemic heart disease (IHD) pathophysiology , types , its causes and treatmentIschemic heart disease (IHD) pathophysiology , types , its causes and treatment
Ischemic heart disease (IHD) pathophysiology , types , its causes and treatment
 
Cerebrovascular Accident
Cerebrovascular AccidentCerebrovascular Accident
Cerebrovascular Accident
 
Mark O
Mark OMark O
Mark O
 
Syncope
SyncopeSyncope
Syncope
 

More from PROFESSOR DR. MD. TOUFIQUR RAHMAN

Hrid Spondon Part 8 dr md toufiqur rahman .pdf
Hrid Spondon Part 8 dr md toufiqur rahman .pdfHrid Spondon Part 8 dr md toufiqur rahman .pdf
Hrid Spondon Part 8 dr md toufiqur rahman .pdf
PROFESSOR DR. MD. TOUFIQUR RAHMAN
 
The Pulmonary Paradox : Navigating the Challenges of Pulmonary Hypertension ...
The Pulmonary Paradox : Navigating the Challenges of Pulmonary Hypertension  ...The Pulmonary Paradox : Navigating the Challenges of Pulmonary Hypertension  ...
The Pulmonary Paradox : Navigating the Challenges of Pulmonary Hypertension ...
PROFESSOR DR. MD. TOUFIQUR RAHMAN
 
Innovations in Cardiology .pdf
Innovations in Cardiology .pdfInnovations in Cardiology .pdf
Innovations in Cardiology .pdf
PROFESSOR DR. MD. TOUFIQUR RAHMAN
 
common cardiac arrhythmias.ppsx
common cardiac arrhythmias.ppsxcommon cardiac arrhythmias.ppsx
common cardiac arrhythmias.ppsx
PROFESSOR DR. MD. TOUFIQUR RAHMAN
 
Rheumatic Heart Disease.ppsx
Rheumatic Heart Disease.ppsxRheumatic Heart Disease.ppsx
Rheumatic Heart Disease.ppsx
PROFESSOR DR. MD. TOUFIQUR RAHMAN
 
Microsoft Excel for medical postgraduates.pdf
Microsoft Excel for medical postgraduates.pdfMicrosoft Excel for medical postgraduates.pdf
Microsoft Excel for medical postgraduates.pdf
PROFESSOR DR. MD. TOUFIQUR RAHMAN
 
Rheumatic fever .pdf
Rheumatic fever .pdfRheumatic fever .pdf
Rheumatic fever .pdf
PROFESSOR DR. MD. TOUFIQUR RAHMAN
 
applied anatomy for undergraduates.pptx
applied anatomy for undergraduates.pptxapplied anatomy for undergraduates.pptx
applied anatomy for undergraduates.pptx
PROFESSOR DR. MD. TOUFIQUR RAHMAN
 
applied physiology for undergraduates.pptx
applied physiology for undergraduates.pptxapplied physiology for undergraduates.pptx
applied physiology for undergraduates.pptx
PROFESSOR DR. MD. TOUFIQUR RAHMAN
 
Peripartum Cardiomyopathy .pdf
Peripartum Cardiomyopathy .pdfPeripartum Cardiomyopathy .pdf
Peripartum Cardiomyopathy .pdf
PROFESSOR DR. MD. TOUFIQUR RAHMAN
 
হৃদ স্পন্দন ৫ম খন্ড .pdf
হৃদ স্পন্দন ৫ম খন্ড .pdfহৃদ স্পন্দন ৫ম খন্ড .pdf
হৃদ স্পন্দন ৫ম খন্ড .pdf
PROFESSOR DR. MD. TOUFIQUR RAHMAN
 
হৃদ স্পন্দন ৬ষ্ঠ খন্ড -সাক্ষাৎকার পর্ব ০১
হৃদ স্পন্দন ৬ষ্ঠ খন্ড -সাক্ষাৎকার পর্ব ০১হৃদ স্পন্দন ৬ষ্ঠ খন্ড -সাক্ষাৎকার পর্ব ০১
হৃদ স্পন্দন ৬ষ্ঠ খন্ড -সাক্ষাৎকার পর্ব ০১
PROFESSOR DR. MD. TOUFIQUR RAHMAN
 
হৃদরোগীদের নিরাপদ দাম্পত্য জীবন.pdf
হৃদরোগীদের নিরাপদ দাম্পত্য জীবন.pdfহৃদরোগীদের নিরাপদ দাম্পত্য জীবন.pdf
হৃদরোগীদের নিরাপদ দাম্পত্য জীবন.pdf
PROFESSOR DR. MD. TOUFIQUR RAHMAN
 
হৃদ স্পন্দন ৭ম খন্ড সাক্ষাৎকার পর্ব ০২.pdf
হৃদ স্পন্দন  ৭ম খন্ড সাক্ষাৎকার পর্ব ০২.pdfহৃদ স্পন্দন  ৭ম খন্ড সাক্ষাৎকার পর্ব ০২.pdf
হৃদ স্পন্দন ৭ম খন্ড সাক্ষাৎকার পর্ব ০২.pdf
PROFESSOR DR. MD. TOUFIQUR RAHMAN
 
উচ্চ রক্তচাপ - নীরব ঘাতক.pdf
উচ্চ রক্তচাপ - নীরব ঘাতক.pdfউচ্চ রক্তচাপ - নীরব ঘাতক.pdf
উচ্চ রক্তচাপ - নীরব ঘাতক.pdf
PROFESSOR DR. MD. TOUFIQUR RAHMAN
 
Cardiac Rehabilatation .pdf
Cardiac Rehabilatation .pdfCardiac Rehabilatation .pdf
Cardiac Rehabilatation .pdf
PROFESSOR DR. MD. TOUFIQUR RAHMAN
 
Manual of basic CPR.pdf
Manual of basic CPR.pdfManual of basic CPR.pdf
Manual of basic CPR.pdf
PROFESSOR DR. MD. TOUFIQUR RAHMAN
 
world Hypertension day 2023.pdf
world Hypertension  day 2023.pdfworld Hypertension  day 2023.pdf
world Hypertension day 2023.pdf
PROFESSOR DR. MD. TOUFIQUR RAHMAN
 
Management of hypertension in elderly .pdf
Management of  hypertension in elderly .pdfManagement of  hypertension in elderly .pdf
Management of hypertension in elderly .pdf
PROFESSOR DR. MD. TOUFIQUR RAHMAN
 
Basic CPR .pptx
Basic CPR .pptxBasic CPR .pptx

More from PROFESSOR DR. MD. TOUFIQUR RAHMAN (20)

Hrid Spondon Part 8 dr md toufiqur rahman .pdf
Hrid Spondon Part 8 dr md toufiqur rahman .pdfHrid Spondon Part 8 dr md toufiqur rahman .pdf
Hrid Spondon Part 8 dr md toufiqur rahman .pdf
 
The Pulmonary Paradox : Navigating the Challenges of Pulmonary Hypertension ...
The Pulmonary Paradox : Navigating the Challenges of Pulmonary Hypertension  ...The Pulmonary Paradox : Navigating the Challenges of Pulmonary Hypertension  ...
The Pulmonary Paradox : Navigating the Challenges of Pulmonary Hypertension ...
 
Innovations in Cardiology .pdf
Innovations in Cardiology .pdfInnovations in Cardiology .pdf
Innovations in Cardiology .pdf
 
common cardiac arrhythmias.ppsx
common cardiac arrhythmias.ppsxcommon cardiac arrhythmias.ppsx
common cardiac arrhythmias.ppsx
 
Rheumatic Heart Disease.ppsx
Rheumatic Heart Disease.ppsxRheumatic Heart Disease.ppsx
Rheumatic Heart Disease.ppsx
 
Microsoft Excel for medical postgraduates.pdf
Microsoft Excel for medical postgraduates.pdfMicrosoft Excel for medical postgraduates.pdf
Microsoft Excel for medical postgraduates.pdf
 
Rheumatic fever .pdf
Rheumatic fever .pdfRheumatic fever .pdf
Rheumatic fever .pdf
 
applied anatomy for undergraduates.pptx
applied anatomy for undergraduates.pptxapplied anatomy for undergraduates.pptx
applied anatomy for undergraduates.pptx
 
applied physiology for undergraduates.pptx
applied physiology for undergraduates.pptxapplied physiology for undergraduates.pptx
applied physiology for undergraduates.pptx
 
Peripartum Cardiomyopathy .pdf
Peripartum Cardiomyopathy .pdfPeripartum Cardiomyopathy .pdf
Peripartum Cardiomyopathy .pdf
 
হৃদ স্পন্দন ৫ম খন্ড .pdf
হৃদ স্পন্দন ৫ম খন্ড .pdfহৃদ স্পন্দন ৫ম খন্ড .pdf
হৃদ স্পন্দন ৫ম খন্ড .pdf
 
হৃদ স্পন্দন ৬ষ্ঠ খন্ড -সাক্ষাৎকার পর্ব ০১
হৃদ স্পন্দন ৬ষ্ঠ খন্ড -সাক্ষাৎকার পর্ব ০১হৃদ স্পন্দন ৬ষ্ঠ খন্ড -সাক্ষাৎকার পর্ব ০১
হৃদ স্পন্দন ৬ষ্ঠ খন্ড -সাক্ষাৎকার পর্ব ০১
 
হৃদরোগীদের নিরাপদ দাম্পত্য জীবন.pdf
হৃদরোগীদের নিরাপদ দাম্পত্য জীবন.pdfহৃদরোগীদের নিরাপদ দাম্পত্য জীবন.pdf
হৃদরোগীদের নিরাপদ দাম্পত্য জীবন.pdf
 
হৃদ স্পন্দন ৭ম খন্ড সাক্ষাৎকার পর্ব ০২.pdf
হৃদ স্পন্দন  ৭ম খন্ড সাক্ষাৎকার পর্ব ০২.pdfহৃদ স্পন্দন  ৭ম খন্ড সাক্ষাৎকার পর্ব ০২.pdf
হৃদ স্পন্দন ৭ম খন্ড সাক্ষাৎকার পর্ব ০২.pdf
 
উচ্চ রক্তচাপ - নীরব ঘাতক.pdf
উচ্চ রক্তচাপ - নীরব ঘাতক.pdfউচ্চ রক্তচাপ - নীরব ঘাতক.pdf
উচ্চ রক্তচাপ - নীরব ঘাতক.pdf
 
Cardiac Rehabilatation .pdf
Cardiac Rehabilatation .pdfCardiac Rehabilatation .pdf
Cardiac Rehabilatation .pdf
 
Manual of basic CPR.pdf
Manual of basic CPR.pdfManual of basic CPR.pdf
Manual of basic CPR.pdf
 
world Hypertension day 2023.pdf
world Hypertension  day 2023.pdfworld Hypertension  day 2023.pdf
world Hypertension day 2023.pdf
 
Management of hypertension in elderly .pdf
Management of  hypertension in elderly .pdfManagement of  hypertension in elderly .pdf
Management of hypertension in elderly .pdf
 
Basic CPR .pptx
Basic CPR .pptxBasic CPR .pptx
Basic CPR .pptx
 

Recently uploaded

Superficial & Deep Fascia of the NECK.pptx
Superficial & Deep Fascia of the NECK.pptxSuperficial & Deep Fascia of the NECK.pptx
Superficial & Deep Fascia of the NECK.pptx
Dr. Rabia Inam Gandapore
 
Novas diretrizes da OMS para os cuidados perinatais de mais qualidade
Novas diretrizes da OMS para os cuidados perinatais de mais qualidadeNovas diretrizes da OMS para os cuidados perinatais de mais qualidade
Novas diretrizes da OMS para os cuidados perinatais de mais qualidade
Prof. Marcus Renato de Carvalho
 
Knee anatomy and clinical tests 2024.pdf
Knee anatomy and clinical tests 2024.pdfKnee anatomy and clinical tests 2024.pdf
Knee anatomy and clinical tests 2024.pdf
vimalpl1234
 
Triangles of Neck and Clinical Correlation by Dr. RIG.pptx
Triangles of Neck and Clinical Correlation by Dr. RIG.pptxTriangles of Neck and Clinical Correlation by Dr. RIG.pptx
Triangles of Neck and Clinical Correlation by Dr. RIG.pptx
Dr. Rabia Inam Gandapore
 
KDIGO 2024 guidelines for diabetologists
KDIGO 2024 guidelines for diabetologistsKDIGO 2024 guidelines for diabetologists
KDIGO 2024 guidelines for diabetologists
د.محمود نجيب
 
heat stroke and heat exhaustion in children
heat stroke and heat exhaustion in childrenheat stroke and heat exhaustion in children
heat stroke and heat exhaustion in children
SumeraAhmad5
 
NVBDCP.pptx Nation vector borne disease control program
NVBDCP.pptx Nation vector borne disease control programNVBDCP.pptx Nation vector borne disease control program
NVBDCP.pptx Nation vector borne disease control program
Sapna Thakur
 
Maxilla, Mandible & Hyoid Bone & Clinical Correlations by Dr. RIG.pptx
Maxilla, Mandible & Hyoid Bone & Clinical Correlations by Dr. RIG.pptxMaxilla, Mandible & Hyoid Bone & Clinical Correlations by Dr. RIG.pptx
Maxilla, Mandible & Hyoid Bone & Clinical Correlations by Dr. RIG.pptx
Dr. Rabia Inam Gandapore
 
BRACHYTHERAPY OVERVIEW AND APPLICATORS
BRACHYTHERAPY OVERVIEW  AND  APPLICATORSBRACHYTHERAPY OVERVIEW  AND  APPLICATORS
BRACHYTHERAPY OVERVIEW AND APPLICATORS
Krishan Murari
 
Flu Vaccine Alert in Bangalore Karnataka
Flu Vaccine Alert in Bangalore KarnatakaFlu Vaccine Alert in Bangalore Karnataka
Flu Vaccine Alert in Bangalore Karnataka
addon Scans
 
24 Upakrama.pptx class ppt useful in all
24 Upakrama.pptx class ppt useful in all24 Upakrama.pptx class ppt useful in all
24 Upakrama.pptx class ppt useful in all
DrSathishMS1
 
TEST BANK for Operations Management, 14th Edition by William J. Stevenson, Ve...
TEST BANK for Operations Management, 14th Edition by William J. Stevenson, Ve...TEST BANK for Operations Management, 14th Edition by William J. Stevenson, Ve...
TEST BANK for Operations Management, 14th Edition by William J. Stevenson, Ve...
kevinkariuki227
 
ARTHROLOGY PPT NCISM SYLLABUS AYURVEDA STUDENTS
ARTHROLOGY PPT NCISM SYLLABUS AYURVEDA STUDENTSARTHROLOGY PPT NCISM SYLLABUS AYURVEDA STUDENTS
ARTHROLOGY PPT NCISM SYLLABUS AYURVEDA STUDENTS
Dr. Vinay Pareek
 
Tom Selleck Health: A Comprehensive Look at the Iconic Actor’s Wellness Journey
Tom Selleck Health: A Comprehensive Look at the Iconic Actor’s Wellness JourneyTom Selleck Health: A Comprehensive Look at the Iconic Actor’s Wellness Journey
Tom Selleck Health: A Comprehensive Look at the Iconic Actor’s Wellness Journey
greendigital
 
Evaluation of antidepressant activity of clitoris ternatea in animals
Evaluation of antidepressant activity of clitoris ternatea in animalsEvaluation of antidepressant activity of clitoris ternatea in animals
Evaluation of antidepressant activity of clitoris ternatea in animals
Shweta
 
micro teaching on communication m.sc nursing.pdf
micro teaching on communication m.sc nursing.pdfmicro teaching on communication m.sc nursing.pdf
micro teaching on communication m.sc nursing.pdf
Anurag Sharma
 
Ocular injury ppt Upendra pal optometrist upums saifai etawah
Ocular injury  ppt  Upendra pal  optometrist upums saifai etawahOcular injury  ppt  Upendra pal  optometrist upums saifai etawah
Ocular injury ppt Upendra pal optometrist upums saifai etawah
pal078100
 
basicmodesofventilation2022-220313203758.pdf
basicmodesofventilation2022-220313203758.pdfbasicmodesofventilation2022-220313203758.pdf
basicmodesofventilation2022-220313203758.pdf
aljamhori teaching hospital
 
BENIGN PROSTATIC HYPERPLASIA.BPH. BPHpdf
BENIGN PROSTATIC HYPERPLASIA.BPH. BPHpdfBENIGN PROSTATIC HYPERPLASIA.BPH. BPHpdf
BENIGN PROSTATIC HYPERPLASIA.BPH. BPHpdf
DR SETH JOTHAM
 
New Directions in Targeted Therapeutic Approaches for Older Adults With Mantl...
New Directions in Targeted Therapeutic Approaches for Older Adults With Mantl...New Directions in Targeted Therapeutic Approaches for Older Adults With Mantl...
New Directions in Targeted Therapeutic Approaches for Older Adults With Mantl...
i3 Health
 

Recently uploaded (20)

Superficial & Deep Fascia of the NECK.pptx
Superficial & Deep Fascia of the NECK.pptxSuperficial & Deep Fascia of the NECK.pptx
Superficial & Deep Fascia of the NECK.pptx
 
Novas diretrizes da OMS para os cuidados perinatais de mais qualidade
Novas diretrizes da OMS para os cuidados perinatais de mais qualidadeNovas diretrizes da OMS para os cuidados perinatais de mais qualidade
Novas diretrizes da OMS para os cuidados perinatais de mais qualidade
 
Knee anatomy and clinical tests 2024.pdf
Knee anatomy and clinical tests 2024.pdfKnee anatomy and clinical tests 2024.pdf
Knee anatomy and clinical tests 2024.pdf
 
Triangles of Neck and Clinical Correlation by Dr. RIG.pptx
Triangles of Neck and Clinical Correlation by Dr. RIG.pptxTriangles of Neck and Clinical Correlation by Dr. RIG.pptx
Triangles of Neck and Clinical Correlation by Dr. RIG.pptx
 
KDIGO 2024 guidelines for diabetologists
KDIGO 2024 guidelines for diabetologistsKDIGO 2024 guidelines for diabetologists
KDIGO 2024 guidelines for diabetologists
 
heat stroke and heat exhaustion in children
heat stroke and heat exhaustion in childrenheat stroke and heat exhaustion in children
heat stroke and heat exhaustion in children
 
NVBDCP.pptx Nation vector borne disease control program
NVBDCP.pptx Nation vector borne disease control programNVBDCP.pptx Nation vector borne disease control program
NVBDCP.pptx Nation vector borne disease control program
 
Maxilla, Mandible & Hyoid Bone & Clinical Correlations by Dr. RIG.pptx
Maxilla, Mandible & Hyoid Bone & Clinical Correlations by Dr. RIG.pptxMaxilla, Mandible & Hyoid Bone & Clinical Correlations by Dr. RIG.pptx
Maxilla, Mandible & Hyoid Bone & Clinical Correlations by Dr. RIG.pptx
 
BRACHYTHERAPY OVERVIEW AND APPLICATORS
BRACHYTHERAPY OVERVIEW  AND  APPLICATORSBRACHYTHERAPY OVERVIEW  AND  APPLICATORS
BRACHYTHERAPY OVERVIEW AND APPLICATORS
 
Flu Vaccine Alert in Bangalore Karnataka
Flu Vaccine Alert in Bangalore KarnatakaFlu Vaccine Alert in Bangalore Karnataka
Flu Vaccine Alert in Bangalore Karnataka
 
24 Upakrama.pptx class ppt useful in all
24 Upakrama.pptx class ppt useful in all24 Upakrama.pptx class ppt useful in all
24 Upakrama.pptx class ppt useful in all
 
TEST BANK for Operations Management, 14th Edition by William J. Stevenson, Ve...
TEST BANK for Operations Management, 14th Edition by William J. Stevenson, Ve...TEST BANK for Operations Management, 14th Edition by William J. Stevenson, Ve...
TEST BANK for Operations Management, 14th Edition by William J. Stevenson, Ve...
 
ARTHROLOGY PPT NCISM SYLLABUS AYURVEDA STUDENTS
ARTHROLOGY PPT NCISM SYLLABUS AYURVEDA STUDENTSARTHROLOGY PPT NCISM SYLLABUS AYURVEDA STUDENTS
ARTHROLOGY PPT NCISM SYLLABUS AYURVEDA STUDENTS
 
Tom Selleck Health: A Comprehensive Look at the Iconic Actor’s Wellness Journey
Tom Selleck Health: A Comprehensive Look at the Iconic Actor’s Wellness JourneyTom Selleck Health: A Comprehensive Look at the Iconic Actor’s Wellness Journey
Tom Selleck Health: A Comprehensive Look at the Iconic Actor’s Wellness Journey
 
Evaluation of antidepressant activity of clitoris ternatea in animals
Evaluation of antidepressant activity of clitoris ternatea in animalsEvaluation of antidepressant activity of clitoris ternatea in animals
Evaluation of antidepressant activity of clitoris ternatea in animals
 
micro teaching on communication m.sc nursing.pdf
micro teaching on communication m.sc nursing.pdfmicro teaching on communication m.sc nursing.pdf
micro teaching on communication m.sc nursing.pdf
 
Ocular injury ppt Upendra pal optometrist upums saifai etawah
Ocular injury  ppt  Upendra pal  optometrist upums saifai etawahOcular injury  ppt  Upendra pal  optometrist upums saifai etawah
Ocular injury ppt Upendra pal optometrist upums saifai etawah
 
basicmodesofventilation2022-220313203758.pdf
basicmodesofventilation2022-220313203758.pdfbasicmodesofventilation2022-220313203758.pdf
basicmodesofventilation2022-220313203758.pdf
 
BENIGN PROSTATIC HYPERPLASIA.BPH. BPHpdf
BENIGN PROSTATIC HYPERPLASIA.BPH. BPHpdfBENIGN PROSTATIC HYPERPLASIA.BPH. BPHpdf
BENIGN PROSTATIC HYPERPLASIA.BPH. BPHpdf
 
New Directions in Targeted Therapeutic Approaches for Older Adults With Mantl...
New Directions in Targeted Therapeutic Approaches for Older Adults With Mantl...New Directions in Targeted Therapeutic Approaches for Older Adults With Mantl...
New Directions in Targeted Therapeutic Approaches for Older Adults With Mantl...
 

MBBS QUESTION ANSWER 3 CARD.pdf

  • 1. Professor DR Md . TOUFIQUR RAHMAN , FCPS, MD Professor & Head, Cardiology, CMMC, Manikganj drtoufiq19711@yahoo.com; drtoufiq1971@gmail.com What are the causes of sinus bradycardia? (DU-04Ja) Sinus bradycardia is a condition where the sinus node in the heart beats slower than the normal range of 60-100 beats per minute. Some common causes of sinus bradycardia include:  Vagal stimulation: This occurs due to an increased activity of the vagus nerve, which is responsible for slowing down the heart rate.  Medications: Certain medications like beta-blockers, calcium channel blockers, and digoxin can cause sinus bradycardia.  Hypothyroidism: Inadequate production of thyroid hormones can cause a decrease in metabolic rate and lead to bradycardia.  Increased intracranial pressure: High pressure within the skull due to conditions like head injury, brain tumors or bleeding can affect the autonomic nervous system and cause bradycardia.  Obstructive sleep apnea: Repeated episodes of apnea during sleep can cause bradycardia due to decreased oxygen supply to the body.  Aging: As the body ages, the electrical activity of the heart can slow down, leading to sinus bradycardia. Other causes of sinus bradycardia include viral infections, genetic disorders, and certain electrolyte imbalances. 2. A 25 years old female presented with palpitation, on examination her pulse was irregularly irregular. How will you assess and investigate her? (DU- 05Ja) The patient's presentation suggests the possibility of atrial fibrillation, which is a common arrhythmia characterized by an irregularly irregular pulse. The following are the steps that can be taken to assess and investigate the patient:  History taking: Obtain a detailed history of the patient's symptoms, including the onset, duration, and frequency of palpitations, associated symptoms, and any relevant medical history.  Physical examination: Conduct a thorough physical examination, including a cardiovascular examination, to assess the patient's heart sounds, rhythm, and rate. Check for any signs of heart failure or underlying heart disease.
  • 2. Professor DR Md . TOUFIQUR RAHMAN , FCPS, MD Professor & Head, Cardiology, CMMC, Manikganj drtoufiq19711@yahoo.com; drtoufiq1971@gmail.com  Electrocardiogram (ECG): Perform an ECG to confirm the diagnosis of atrial fibrillation and to determine the heart rate and rhythm. An ECG will also help rule out other arrhythmias or underlying heart conditions.  Blood tests: Check the patient's thyroid function, electrolyte levels, and other relevant blood tests to identify any underlying conditions that may be causing the arrhythmia.  Echocardiography: Perform an echocardiogram to assess the structure and function of the heart and to identify any underlying heart disease.  Holter monitor: Use a Holter monitor to monitor the patient's heart rate and rhythm over a 24- hour period to identify any episodes of atrial fibrillation that may not be captured during a routine ECG. Other tests: Consider other tests, such as a stress test or electrophysiology study, if necessary, to further evaluate the patient's heart function and arrhythmia. In summary, the assessment and investigation of a patient with palpitations and an irregularly irregular pulse would involve a comprehensive evaluation of the patient's symptoms, physical examination, ECG, blood tests, echocardiogram, and possibly a Holter monitor or other tests to confirm the diagnosis and identify any underlying conditions. 3. Write short note on : Stokes Adams syndrome. (DU- 06M) Stokes-Adams syndrome, also known as complete heart block, is a condition characterized by episodes of syncope (fainting) or seizures due to a sudden and temporary interruption of the heart's electrical conduction system. The underlying cause of Stokes-Adams syndrome is often attributed to degeneration or blockage of the bundle of His, which is a crucial part of the heart's conduction system responsible for transmitting electrical impulses from the atria to the ventricles. This can lead to a slower heart rate or complete blockage of the electrical impulses to the ventricles. Symptoms of Stokes-Adams syndrome can include dizziness, lightheadedness, loss of consciousness, and seizures. Treatment may involve the use of a pacemaker to regulate the heart's electrical impulses or medication to manage symptoms. In severe cases, surgical intervention may be required.
  • 3. Professor DR Md . TOUFIQUR RAHMAN , FCPS, MD Professor & Head, Cardiology, CMMC, Manikganj drtoufiq19711@yahoo.com; drtoufiq1971@gmail.com 3. Write short note on : Stokes Adams syndrome. (DU- 06M) Stokes-Adams syndrome is a medical condition characterized by episodes of fainting or syncope, caused by a sudden drop in heart rate due to a disturbance in the heart's electrical conduction system. Here are some key points about Stokes-Adams syndrome: The syndrome is named after two physicians, William Stokes and Robert Adams, who first described the condition in 1828 and 1827, respectively. Stokes-Adams syndrome is caused by a blockage or interruption in the electrical signals that regulate the heartbeat, leading to an abnormally slow heart rate (bradycardia) or even a complete pause in the heart's rhythm (asystole). The most common underlying cause of Stokes-Adams syndrome is a type of heart block known as atrioventricular (AV) block, which occurs when the electrical signals that travel from the atria to the ventricles are slowed or blocked. Symptoms of Stokes-Adams syndrome can include dizziness, lightheadedness, fainting, confusion, chest pain, and shortness of breath. Treatment of Stokes-Adams syndrome depends on the severity of the symptoms and the underlying cause of the heart block. Mild cases may be managed with medications such as atropine or isoproterenol to increase heart rate, while more severe cases may require the implantation of a pacemaker to regulate the heart's rhythm. Complications of Stokes-Adams syndrome can include falls and injuries from fainting, as well as more serious complications such as stroke or sudden cardiac arrest. Angina Pectoris 1. What are the clinical menifestations of coronary heart disease ? (DU-09Ja) Coronary heart disease (CHD) is a condition caused by the narrowing or blockage of the coronary arteries, which supply blood and oxygen to the heart muscle. The clinical manifestations of CHD can vary widely depending on the degree and location of coronary artery involvement. Some common clinical manifestations of CHD include:  Chest pain or discomfort: Angina pectoris is the most common symptom of CHD. The pain is usually described as a tightness, pressure, squeezing, or burning sensation in the chest. It may also be felt in the arms, neck, jaw, shoulder, or back.  Shortness of breath: This symptom may occur with or without chest pain. It can be caused by the heart not getting enough oxygen due to reduced blood flow.
  • 4. Professor DR Md . TOUFIQUR RAHMAN , FCPS, MD Professor & Head, Cardiology, CMMC, Manikganj drtoufiq19711@yahoo.com; drtoufiq1971@gmail.com  Palpitations: This is a sensation of rapid or irregular heartbeat. It may feel like the heart is skipping a beat or beating too fast.  Fatigue: Reduced blood flow to the heart can cause fatigue or weakness.  Dizziness or lightheadedness: Reduced blood flow to the brain can cause dizziness or lightheadedness.  Syncope: Fainting may occur due to decreased blood flow to the brain. Other symptoms: Nausea, vomiting, sweating, and anxiety may also occur. It is important to note that some people with CHD may not experience any symptoms, especially in the early stages of the disease. 2. Describe a typical angina. (DU- 07S) A typical angina, also known as stable angina or exertional angina, is a type of chest pain that occurs when the heart muscle does not get enough oxygenated blood due to narrowed coronary arteries. The chest pain is usually described as a squeezing, pressure, heaviness, or tightness in the chest, which may radiate to the left arm, neck, jaw, shoulder, or back. It is often triggered by physical exertion or emotional stress and relieved by rest or sublingual nitroglycerin. The pain usually lasts for a few minutes (up to 10 minutes) and is not associated with shortness of breath, nausea, or diaphoresis. Patients with typical angina may have a history of atherosclerotic risk factors such as hypertension, diabetes mellitus, hyperlipidemia, or smoking. 2. Describe a typical angina. (DU- 07S) Typical angina is a clinical manifestation of coronary artery disease. It is characterized by chest pain or discomfort that occurs with exertion, emotion, or other types of stress and is relieved by rest or nitroglycerin. The following are the typical characteristics of angina:  Location: The pain is typically felt behind the sternum or breastbone, but can also be felt in the left arm, neck, jaw, or shoulders.  Quality: The pain is described as a squeezing, pressure, heaviness, or tightness sensation.  Duration: The pain usually lasts from a few seconds to a few minutes and is relieved by rest or nitroglycerin.  Triggers: The pain is typically triggered by physical exertion, emotional stress, exposure to cold temperatures, or after a heavy meal.
  • 5. Professor DR Md . TOUFIQUR RAHMAN , FCPS, MD Professor & Head, Cardiology, CMMC, Manikganj drtoufiq19711@yahoo.com; drtoufiq1971@gmail.com  Response to nitroglycerin: The pain is relieved by nitroglycerin, a medication that dilates the coronary arteries, allowing more blood flow to the heart muscle. These characteristics are used to differentiate typical angina from other types of chest pain, such as atypical angina, unstable angina, or myocardial infarction. 3. How will you clinical differentiate angina pectoris from esophageal pain? (DU- 06Ja) Differentiating angina pectoris from esophageal pain can be challenging because both can present with similar symptoms such as retrosternal discomfort or burning pain. However, there are some clinical differences that can help to distinguish between the two conditions:  Onset and duration of pain: Angina pectoris typically presents with sudden onset of pain that lasts for a few minutes (2-5 minutes), whereas esophageal pain often has a gradual onset and lasts for a longer duration.  Provoking factors: Angina pectoris is usually provoked by physical or emotional stress, whereas esophageal pain is often provoked by ingestion of food or acid reflux.  Radiation of pain: Angina pectoris commonly radiates to the left arm, shoulder, neck, jaw or back, while esophageal pain typically radiates to the back, neck or arms.  Response to nitroglycerin: Nitroglycerin can relieve the symptoms of angina pectoris, but has no effect on esophageal pain.  Associated symptoms: Angina pectoris may be associated with shortness of breath, sweating or nausea, while esophageal pain may be associated with heartburn, regurgitation or difficulty swallowing. Therefore, taking a detailed history of the patient's symptoms, including the onset, duration, provoking factors, radiation of pain, and associated symptoms, can help to differentiate angina pectoris from esophageal pain. In addition, performing a physical examination and specific tests such as an electrocardiogram (ECG) or an esophageal pH monitoring can also aid in the diagnosis. How would you investigate a patient with stable angina with expected findings? (DU-19Nov) The investigation of a patient with stable angina typically involves the following:
  • 6. Professor DR Md . TOUFIQUR RAHMAN , FCPS, MD Professor & Head, Cardiology, CMMC, Manikganj drtoufiq19711@yahoo.com; drtoufiq1971@gmail.com  Electrocardiogram (ECG): A resting ECG may show evidence of previous myocardial infarction (MI) or ischemia. During an angina episode, ECG changes may be present, such as ST- segment depression or T-wave inversion.  Exercise stress test: A stress test is a commonly used diagnostic test for stable angina. It involves monitoring the ECG while the patient exercises, usually on a treadmill. The test helps to identify ischemia-induced changes in the ECG and to determine the patient's exercise capacity.  Echocardiography: This non-invasive imaging test uses sound waves to visualize the heart and its function. It can help identify structural abnormalities, assess the function of the heart valves, and estimate the ejection fraction (a measure of the heart's pumping efficiency).  Coronary angiography: This is an invasive diagnostic test that involves injecting a contrast dye into the coronary arteries and taking X-ray images to visualize any blockages or narrowing in the arteries. It is usually reserved for patients with high-risk features or those who may require invasive treatments such as angioplasty or bypass surgery.  Blood tests: Cardiac biomarkers such as troponin may be measured to rule out acute coronary syndrome as the cause of symptoms. The expected findings of these investigations depend on the severity and extent of the coronary artery disease. In stable angina, the ECG may show evidence of previous MI or ischemia, and the exercise stress test may show evidence of ischemia-induced ECG changes. Echocardiography may show reduced left ventricular function, and coronary angiography may reveal areas of narrowing or blockages in the coronary arteries. Blood tests such as troponin are usually normal in stable angina. In addition to investigating for stable angina, it is also important to investigate for risk factors that may contribute to the development of coronary artery disease and unstable angina. Some of the investigations that may be done include:  Lipid profile: This measures the levels of cholesterol and other fats in the blood, which can contribute to the development of atherosclerosis and subsequent angina.  Fasting blood glucose: This test measures the amount of glucose in the blood after a period of fasting. Elevated levels can indicate the presence of diabetes, which is a risk factor for coronary artery disease and angina.  Blood pressure: Elevated blood pressure is a risk factor for the development of coronary artery disease and angina.
  • 7. Professor DR Md . TOUFIQUR RAHMAN , FCPS, MD Professor & Head, Cardiology, CMMC, Manikganj drtoufiq19711@yahoo.com; drtoufiq1971@gmail.com  ECG: This test can identify any abnormalities in the heart's electrical activity, which can indicate the presence of coronary artery disease and angina.  Stress test: This test involves monitoring the heart's response to physical activity or medication- induced stress, and can help diagnose the presence of coronary artery disease and angina.  Coronary angiography: This is an invasive test that involves injecting dye into the coronary arteries to visualize any blockages or narrowing in the vessels. This test is typically reserved for patients with suspected or confirmed unstable angina. 5. Write down the treatment of angina pectoris (DU-06Ja) The treatment of angina pectoris may include:  Lifestyle modifications: The patient may be advised to quit smoking, maintain a healthy weight, exercise regularly, and follow a healthy diet.  Medications: Medications such as nitrates, beta-blockers, calcium channel blockers, and antiplatelet agents may be prescribed to control symptoms, reduce the risk of complications and improve outcomes.  Revascularization: Revascularization procedures such as angioplasty or coronary artery bypass surgery may be recommended in some cases to improve blood flow to the heart and relieve symptoms.  Education and counseling: Patients with angina should be educated about their condition, advised to recognize and avoid triggers, and counseled about stress reduction techniques.  Follow-up care: Patients with angina require regular follow-up care to monitor their symptoms, medications, and risk factors. What do you mean by unstable angina? (DU- 07S) How would you manage a case of unstable angina? (DU- 05Ja) Unstable angina is a type of chest pain or discomfort that occurs when a person is at rest or experiencing minimal exertion and is considered a medical emergency. It is caused by the disruption of atherosclerotic plaques in the coronary arteries, leading to partial or complete blockage of blood flow to the heart muscle. Management of unstable angina involves immediate hospitalization and treatment to reduce the risk of heart attack or cardiac arrest. The following steps can be taken:
  • 8. Professor DR Md . TOUFIQUR RAHMAN , FCPS, MD Professor & Head, Cardiology, CMMC, Manikganj drtoufiq19711@yahoo.com; drtoufiq1971@gmail.com  Oxygen therapy to increase the oxygen supply to the heart  Nitroglycerin to relieve chest pain by dilating the blood vessels  Aspirin or other antiplatelet agents to prevent the formation of blood clots  Heparin or other anticoagulant medications to prevent the formation of blood clots  Beta-blockers to reduce the workload on the heart and lower blood pressure  Statins to lower cholesterol levels and reduce the risk of future cardiovascular events  Invasive procedures such as angiography and percutaneous coronary intervention (PCI) or coronary artery bypass grafting (CABG) may be necessary to treat underlying coronary artery disease and prevent future episodes of unstable angina or myocardial infarction.  The specific treatment plan will depend on the severity of the condition and individual patient factors, and should be determined by a healthcare provider with expertise in managing unstable angina. Myocardial Infarction (MI) *** 1. (a) How would you manage a case of acute myocardial infarction? (DU- 20M) (a) Management of a case of acute myocardial infarction typically involves the following steps:  Rapid assessment and stabilization of the patient's condition: The patient's vital signs and cardiac rhythm should be monitored, and interventions should be made to stabilize them as needed. Oxygen should be administered to maintain oxygen saturation levels above 94%. Nitroglycerin and morphine may be given to relieve pain and reduce anxiety.  Early reperfusion therapy: The goal of reperfusion therapy is to restore blood flow to the affected area of the heart as quickly as possible. This can be achieved through percutaneous coronary intervention (PCI) or fibrinolytic therapy, depending on the patient's clinical presentation and the availability of resources.  Antiplatelet and anticoagulant therapy: Antiplatelet therapy with aspirin should be initiated as soon as possible. Additional anticoagulant therapy with heparin or low- molecular-weight heparin may also be given.  Beta-blocker therapy: Beta-blockers should be started within the first 24 hours of an acute myocardial infarction to reduce the risk of subsequent events.
  • 9. Professor DR Md . TOUFIQUR RAHMAN , FCPS, MD Professor & Head, Cardiology, CMMC, Manikganj drtoufiq19711@yahoo.com; drtoufiq1971@gmail.com  Statin therapy: Statins should be initiated in all patients with acute myocardial infarction to reduce the risk of recurrent events.  Cardiac rehabilitation: Cardiac rehabilitation should be started as soon as possible to help the patient recover from the acute event and reduce the risk of future cardiac events. (b) The management of a case of acute myocardial infarction should be tailored to the individual patient's clinical presentation and underlying risk factors. The specific interventions and therapies used will depend on the patient's hemodynamic stability, the presence of complications, and the availability of resources. In general, the goal of treatment is to restore blood flow to the affected area of the heart, minimize myocardial damage, and prevent subsequent events. Close monitoring and prompt intervention are essential to optimize outcomes. (b) What are early complications of acute myocardial infarction? (DU- M) Some of the early complications of acute myocardial infarction (AMI) include: Arrhythmias: It is a common complication of AMI and can occur due to damage to the electrical conduction system of the heart. Arrhythmias can be life-threatening and include ventricular tachycardia, ventricular fibrillation, and heart block. Heart failure: AMI can damage a significant portion of the heart muscle, leading to decreased cardiac function and heart failure. Cardiogenic shock: In severe cases, AMI can cause cardiogenic shock, which occurs when the heart is unable to pump enough blood to meet the body's demands. Pericarditis: Inflammation of the pericardium, the sac surrounding the heart, can occur after an AMI. This can cause chest pain and other symptoms. Myocardial rupture: Rarely, the infarcted area of the heart muscle can rupture, leading to severe internal bleeding and shock.
  • 10. Professor DR Md . TOUFIQUR RAHMAN , FCPS, MD Professor & Head, Cardiology, CMMC, Manikganj drtoufiq19711@yahoo.com; drtoufiq1971@gmail.com Ventricular septal defect: AMI can cause a hole to form in the wall that separates the two lower chambers of the heart, leading to significant hemodynamic compromise. Mitral regurgitation: Damage to the papillary muscles or the valve itself can lead to mitral regurgitation, which can cause heart failure and other complications. Thromboembolic events: Blood clots can form within the heart or in the blood vessels leading to or from the heart, leading to stroke or other thromboembolic events. Mention the principles of management of acute myocardial infarction. (DU- 19M) The principles of management of acute myocardial infarction include:  Early recognition and diagnosis: Prompt recognition of MI symptoms and confirmation through diagnostic tests such as electrocardiogram (ECG), cardiac biomarkers (troponin, CK- MB), and imaging studies (echocardiography, coronary angiography).  Reperfusion therapy: Restoration of blood flow to the ischemic myocardium through either primary percutaneous coronary intervention (PCI) or fibrinolytic therapy. The choice of therapy depends on the availability, time to treatment, and patient's clinical characteristics.  Pharmacological therapy: Administration of antiplatelet agents, anticoagulants, beta- blockers, angiotensin-converting enzyme inhibitors (ACEIs), angiotensin receptor blockers (ARBs), and lipid-lowering agents such as statins.  Monitoring and supportive care: Continuous monitoring of vital signs, ECG, and cardiac biomarkers for any signs of worsening or complications such as arrhythmias, heart failure, or cardiogenic shock. Oxygen therapy, pain relief, and hemodynamic support may also be necessary.  Risk factor modification and secondary prevention: Implementation of lifestyle modifications such as smoking cessation, healthy diet, regular exercise, and stress reduction, as well as initiation of medications for secondary prevention of cardiovascular events such as aspirin, statins, and ACEIs/ARBs.  Cardiac rehabilitation: A structured program of exercise training, risk factor modification, and psychosocial support to improve functional capacity and quality of life after an acute MI.
  • 11. Professor DR Md . TOUFIQUR RAHMAN , FCPS, MD Professor & Head, Cardiology, CMMC, Manikganj drtoufiq19711@yahoo.com; drtoufiq1971@gmail.com Describe the characteristics of chest pain of acute myocardial infarction. (DU-18Nov) The characteristics of chest pain in acute myocardial infarction (AMI) are as follows:  Location: The pain is typically felt in the center of the chest behind the sternum, although it can also radiate to the left arm, neck, jaw, back, or epigastrium.  Quality: The pain is usually described as a pressure, squeezing, tightness, or burning sensation. It is often compared to a feeling of an elephant sitting on the chest or a heavy weight.  Severity: The pain is usually severe and intense, often described as the worst pain ever experienced.  Duration: The pain lasts for more than 20 minutes, but can last for several hours or longer.  Associated symptoms: The pain is often associated with other symptoms such as sweating, shortness of breath, nausea, vomiting, dizziness, and palpitations.  Trigger factors: The pain may be triggered by physical or emotional stress, or may occur at rest. It is important to note that not all patients with AMI will experience chest pain, especially in the elderly, women, and patients with diabetes. Therefore, other symptoms such as dyspnea, diaphoresis, nausea, or vomiting may be the presenting features of AMI. 3. A 55 year old man has presented with central chest pain precipitating with physical excretion & cold exposure, relieving with taking rest for 3 months. On precordium auscultation, there is no murmur. (DU-21M) a. What is the most likely clinical diagnosis? Mention 2 (two) other features to differentiate from musculoskeletal chest pain. b. Write down the risk factors of the above condition.
  • 12. Professor DR Md . TOUFIQUR RAHMAN , FCPS, MD Professor & Head, Cardiology, CMMC, Manikganj drtoufiq19711@yahoo.com; drtoufiq1971@gmail.com a. The most likely clinical diagnosis is stable angina. Two features to differentiate from musculoskeletal chest pain are: The pain is typically precipitated by physical exertion or emotional stress and relieved by rest or sublingual nitroglycerin. The pain is described as a squeezing, pressure, heaviness, or tightness sensation in the chest, often with radiation to the left arm, neck, jaw, or back. b. The risk factors of stable angina include:  Age (more common in older adults)  Male gender  Smoking  High blood pressure  High cholesterol levels  Diabetes  Family history of premature coronary artery disease  Obesity or overweight  Sedentary lifestyle 4. a. A 45 year old male presents with sudden severe chest pain for 6 hours and ECG evidence is in favour of acute myocardial infarction. How will you search for underlying risk factors? In a patient with acute myocardial infarction, it is important to search for underlying risk factors that may contribute to the development of the condition and affect the management plan. Some of the common risk factors for myocardial infarction include:  Age: Older age increases the risk of developing myocardial infarction.  Family history of heart disease: Individuals with a family history of heart disease are at a higher risk of developing myocardial infarction.  Smoking: Smoking is a major risk factor for myocardial infarction.  High blood pressure: Individuals with high blood pressure are at an increased risk of myocardial infarction.  High cholesterol: High levels of cholesterol in the blood can increase the risk of myocardial infarction.  Diabetes: Individuals with diabetes are at a higher risk of developing myocardial infarction.  Obesity: Being overweight or obese increases the risk of myocardial infarction.
  • 13. Professor DR Md . TOUFIQUR RAHMAN , FCPS, MD Professor & Head, Cardiology, CMMC, Manikganj drtoufiq19711@yahoo.com; drtoufiq1971@gmail.com  Sedentary lifestyle: Lack of physical activity can increase the risk of myocardial infarction.  Stress: Chronic stress can increase the risk of developing myocardial infarction. To search for underlying risk factors in a patient with acute myocardial infarction, a detailed history and physical examination should be performed. This may include asking about family history, smoking history, dietary habits, exercise habits, and medical history (such as diabetes or hypertension). Blood tests, such as lipid profile and glucose level, can also be performed to assess for underlying risk factors. b. Mention the clinical conditions related to excess environment heat. (DU-17Ja) Clinical conditions related to excess environmental heat include:  Heat cramps: painful muscle spasms that occur during or after intense exercise in hot environments  Heat exhaustion: a condition characterized by heavy sweating, weakness, dizziness, nausea, and headache due to dehydration and electrolyte imbalance  Heat syncope: fainting or dizziness caused by a sudden drop in blood pressure due to standing for prolonged periods in a hot environment  Heat rash: small red bumps or blisters on the skin due to sweating and clogged sweat ducts  Heatstroke: a life-threatening condition characterized by a body temperature of 104°F or higher, confusion, seizures, and loss of consciousness. It is considered a medical emergency and requires immediate treatment. 5. A 60 year old smoker is admitted with acute central chest pain for 2 hours. His ECG shows ST elevation. (DU-16Ja) a. What immediate management would you take for the patient? The immediate management for the patient with acute central chest pain and ST elevation on ECG would be to initiate reperfusion therapy as soon as possible. This can be done with either fibrinolytic therapy or primary percutaneous coronary intervention (PCI) depending on the available resources and the time since the onset of symptoms. Other immediate management measures would include administering oxygen to maintain oxygen saturation above 94%, initiating antiplatelet therapy with aspirin and P2Y12 inhibitors, and providing pain relief with intravenous morphine. The patient should also be closely monitored for complications such as ventricular arrhythmias and cardiogenic shock
  • 14. Professor DR Md . TOUFIQUR RAHMAN , FCPS, MD Professor & Head, Cardiology, CMMC, Manikganj drtoufiq19711@yahoo.com; drtoufiq1971@gmail.com  Call for emergency medical services to transfer the patient to the nearest hospital with a cardiac catheterization laboratory and 24-hour primary percutaneous coronary intervention (PCI) capabilities.  Administer aspirin chewable tablet of 300 mg or 4 tablets of 75 mg each orally, and nitroglycerin sublingual tablet or spray every 5 minutes up to three doses as needed to relieve chest pain.  Initiate oxygen therapy with a target saturation level of at least 94%.  Administer a loading dose of a P2Y12 inhibitor (e.g., clopidogrel 600 mg, prasugrel 60 mg, or ticagrelor 180 mg) orally or through a nasogastric tube.  Perform an electrocardiogram (ECG) and repeat it every 10-15 minutes if there is persistent chest pain.  Obtain venous access and start an intravenous infusion of heparin or bivalirudin, a direct thrombin inhibitor.  Monitor blood pressure, heart rate, oxygen saturation, and ECG changes continuously.  Consider providing analgesia with opioids if chest pain persists despite the administration of nitroglycerin.  Reevaluate the patient and consider the need for urgent revascularization by primary PCI or thrombolytic therapy in consultation with a cardiologist. It is crucial to remember that the management of acute myocardial infarction is time-sensitive, and early recognition and prompt treatment can save lives and prevent complications. b. The patient develops dyspnoea on the following day. What additional measures will you take to manage the patient? If the patient develops dyspnea following an acute myocardial infarction, it may suggest the development of heart failure or a complication such as a ventricular septal defect. The following additional measures may be taken to manage the patient:  Administer supplemental oxygen to improve oxygen saturation levels and relieve dyspnea.  Administer diuretics to reduce fluid overload and relieve dyspnea associated with heart failure.  Administer nitrates to dilate blood vessels and reduce cardiac workload, thereby relieving dyspnea.  Monitor vital signs, fluid balance, and urine output to assess the patient's response to treatment and detect any signs of worsening heart failure.
  • 15. Professor DR Md . TOUFIQUR RAHMAN , FCPS, MD Professor & Head, Cardiology, CMMC, Manikganj drtoufiq19711@yahoo.com; drtoufiq1971@gmail.com  Consider further investigations such as echocardiography to assess the patient's left ventricular function and identify any complications such as ventricular septal defect.  Consult with a cardiologist for further management and possible interventions, such as cardiac catheterization or coronary artery bypass graft surgery, depending on the underlying cause of the dyspnea. 6. a. What is acute coronary syndrome? How do confirm acute myocardial infraction? (DU-15Ju) Acute coronary syndrome (ACS) refers to a group of clinical conditions that are caused by a sudden reduction or occlusion of blood flow to the heart due to coronary artery disease. The three clinical conditions that make up ACS are unstable angina, non-ST segment elevation myocardial infarction (NSTEMI), and ST-segment elevation myocardial infarction (STEMI). To confirm acute myocardial infarction (AMI), several diagnostic tests can be used, including:  Electrocardiogram (ECG): An ECG measures the electrical activity of the heart and can detect changes in the heart's rhythm and electrical conduction. In AMI, the ECG may show ST-segment elevation, indicating myocardial injury.  Cardiac biomarkers: These are blood tests that measure the levels of enzymes and proteins released by damaged heart muscle cells. Elevated levels of cardiac biomarkers such as troponin indicate myocardial damage and help to confirm the diagnosis of AMI.  Imaging studies: Imaging studies such as echocardiography or cardiac MRI may be used to assess the extent and severity of myocardial damage and to help guide treatment decisions. b. What is the difference between NSTEMI and STEMI? (DU-15Ju) The main difference between NSTEMI and STEMI is the extent and severity of myocardial damage. In NSTEMI, the blood flow to the heart is partially occluded, and there is partial myocardial damage. In contrast, in STEMI, there is complete occlusion of the coronary artery, leading to complete myocardial damage.
  • 16. Professor DR Md . TOUFIQUR RAHMAN , FCPS, MD Professor & Head, Cardiology, CMMC, Manikganj drtoufiq19711@yahoo.com; drtoufiq1971@gmail.com Other differences between NSTEMI and STEMI include:  ECG changes: In NSTEMI, the ECG may show ST-segment depression or T-wave inversion, indicating myocardial ischemia. In STEMI, the ECG typically shows ST- segment elevation, indicating myocardial injury.  Cardiac biomarkers: In both NSTEMI and STEMI, cardiac biomarkers such as troponin are elevated, but the levels are typically higher in STEMI due to the more extensive myocardial damage.  Treatment: The treatment of NSTEMI and STEMI differs, with STEMI requiring more urgent and aggressive intervention, such as reperfusion therapy, to restore blood flow to the affected area of the heart. NSTEMI is usually managed with medical therapy and may require less urgent intervention. b. Mention immediate management of this condition. Mention the principles of management of acute coronary syndrome (DU-18Ju) Immediate management of acute coronary syndrome includes:  Administering aspirin: Aspirin is a blood thinner and helps in preventing the formation of blood clots. It should be chewed or crushed and swallowed immediately after a heart attack is suspected.  Administering nitroglycerin: Nitroglycerin helps in relaxing the blood vessels and easing the workload on the heart. It should be given sublingually (under the tongue) or intravenously.  Administering morphine: Morphine helps in relieving the pain associated with acute coronary syndrome. It also reduces the workload on the heart.  Supplemental oxygen: Oxygen should be given to the patient to ensure adequate oxygen supply to the heart.  Beta-blockers: Beta-blockers help in reducing the workload on the heart and are given orally or intravenously. The principles of management of acute coronary syndrome include:
  • 17. Professor DR Md . TOUFIQUR RAHMAN , FCPS, MD Professor & Head, Cardiology, CMMC, Manikganj drtoufiq19711@yahoo.com; drtoufiq1971@gmail.com  Early recognition: Acute coronary syndrome should be recognized as early as possible, and treatment should be started immediately.  Prompt revascularization: Revascularization, either through angioplasty or bypass surgery, should be done as soon as possible to restore blood flow to the affected area of the heart.  Pharmacological therapy: Medications such as aspirin, nitroglycerin, beta-blockers, and statins should be given to manage the symptoms and reduce the risk of future cardiovascular events.  Risk factor management: Risk factors such as hypertension, diabetes, and high cholesterol should be managed to prevent future cardiovascular events.  Cardiac rehabilitation: Cardiac rehabilitation, including physical exercise, lifestyle modifications, and stress management, should be initiated to improve the patient's overall cardiovascular health and prevent future events. 7. Name the enzymes those may be elevated in acute MI. (DU-09Ja) In the context of acute MI, the following enzymes may be elevated:  Creatine kinase (CK-MB): CK-MB levels rise 3-6 hours after MI onset and peak at 12-24 hours. It returns to normal within 2-3 days.  Troponin I or T: Troponin levels rise 3-6 hours after MI onset and remain elevated for up to 2 weeks.  Myoglobin: Myoglobin levels rise within 2-3 hours after MI onset and peak within 6-9 hours. It returns to normal within 24 hours. Elevation of these enzymes in the appropriate clinical context can help confirm the diagnosis of acute MI. 8. Write down the management of acute MI (DU-10Ju) Management of Acute Myocardial Infarction (MI) involves the following steps:  Initial assessment and stabilization: This includes monitoring vital signs, administering oxygen therapy, establishing IV access, providing pain relief with medications such as nitroglycerin and morphine, and assessing the need for urgent reperfusion therapy.  Reperfusion therapy:
  • 18. Professor DR Md . TOUFIQUR RAHMAN , FCPS, MD Professor & Head, Cardiology, CMMC, Manikganj drtoufiq19711@yahoo.com; drtoufiq1971@gmail.com The aim of reperfusion therapy is to restore blood flow to the blocked coronary artery as soon as possible. It can be achieved either by primary percutaneous coronary intervention (PCI) or thrombolytic therapy. Primary PCI is preferred if it can be performed within 90 minutes of first medical contact.  Antiplatelet and anticoagulant therapy: Aspirin is given immediately and continued indefinitely. Other antiplatelet agents such as clopidogrel or ticagrelor are also given. Anticoagulant therapy with heparin or enoxaparin is started and continued for several days.  Beta-blocker therapy: Beta-blockers are given to reduce the risk of recurrent MI, arrhythmias, and heart failure. They should be started as soon as possible and continued indefinitely.  ACE inhibitors/ARBs: These drugs are given to reduce the risk of recurrent MI, heart failure, and death. They should be started within the first 24 hours of MI and continued indefinitely.  Statin therapy: Statins are given to reduce LDL cholesterol levels and decrease the risk of recurrent MI and death. They should be started as soon as possible and continued indefinitely.  Cardiac rehabilitation: This involves exercise training, lifestyle modification, and risk factor reduction. It should be started as soon as possible after discharge and continued for several weeks to months.  Complication management: Management of complications such as heart failure, cardiogenic shock, arrhythmias, and mechanical complications such as papillary muscle rupture or ventricular septal defect should be done promptly.  Long-term follow-up: Patients with acute MI should be followed up regularly for several years to monitor for recurrent events and optimize secondary prevention measures. 9. A 45 year old male presents with sudden severe chest pain for 1 hour. (DU- 19Ja) a. Make a check list history and physical signs to find out the causes. b. How would you manage this case? a. Check list history and physical signs to find out the causes of sudden severe chest pain in a 45-year- old male: History:  Time of onset of pain  Duration and frequency of pain
  • 19. Professor DR Md . TOUFIQUR RAHMAN , FCPS, MD Professor & Head, Cardiology, CMMC, Manikganj drtoufiq19711@yahoo.com; drtoufiq1971@gmail.com  Quality and intensity of pain  Radiation of pain  Any associated symptoms like sweating, nausea, vomiting, breathlessness, palpitation, dizziness, or syncope  Any past history of similar episodes  Any past medical history of hypertension, diabetes mellitus, dyslipidemia, smoking, or family history of coronary artery disease Physical examination:  Vital signs: blood pressure, heart rate, respiratory rate, and oxygen saturation  Cardiac examination: rhythm, rate, murmurs, and gallops  Respiratory examination: breath sounds, crepitations, or wheezing  Abdominal examination: tenderness, guarding, or distension  Neurological examination: level of consciousness, pupils, and motor power b. Management of a case with sudden severe chest pain for 1 hour:  Assess the ABCs (airway, breathing, and circulation)  Administer oxygen via a nasal cannula at a flow rate of 4-6 L/min  Administer aspirin 300mg orally or 150-300mg chewed and swallowed as soon as possible  Administer sublingual glyceryl trinitrate (GTN) 0.4 mg every 5 minutes up to a maximum of 3 doses, if there is no contraindication  Obtain a 12-lead electrocardiogram (ECG) as soon as possible  Consider early reperfusion therapy if indicated (thrombolytic therapy or primary percutaneous coronary intervention) within 12 hours of the onset of symptoms  Administer morphine sulfate 2-4mg intravenously, if there is no contraindication and the pain persists after treatment with GTN  Monitor the patient's vital signs and cardiac rhythm closely  Admit the patient to the coronary care unit for further management and observation 10. A 60 year old man has been admitted with severe central chest pain. His ECG shows ST elevation. (DU-14Ju) a. How do you manage the patient in a Medical College Hospital? b. The patient developed dyspnoea on the following day. What additional measures will you take to manage the patient? a. The management of the patient with severe central chest pain and ST elevation on ECG in a Medical College Hospital typically involves the following steps:
  • 20. Professor DR Md . TOUFIQUR RAHMAN , FCPS, MD Professor & Head, Cardiology, CMMC, Manikganj drtoufiq19711@yahoo.com; drtoufiq1971@gmail.com  Rapid assessment: The patient should be assessed quickly to determine the severity of their condition and the need for urgent intervention.  Oxygen therapy: Oxygen should be administered immediately to improve oxygenation.  Pain relief: Analgesics such as morphine sulfate or fentanyl should be given for pain relief.  Antiplatelet therapy: Aspirin should be given as soon as possible to reduce the risk of further cardiac events.  Reperfusion therapy: If available, reperfusion therapy (such as thrombolysis or primary percutaneous coronary intervention) should be initiated within the recommended time frame to restore blood flow to the affected area of the heart.  Monitoring: The patient should be monitored closely for any changes in their condition, including vital signs, ECG, and cardiac enzymes.  Risk factor management: The patient should receive counseling and interventions to manage their risk factors for cardiovascular disease, such as smoking cessation, blood pressure control, and lipid management. b. If the patient develops dyspnea on the following day after an acute myocardial infarction, additional measures may be needed to manage their condition, including:  Oxygen therapy: Supplemental oxygen should be given to maintain oxygen saturation levels.  Diuretics: Diuretics may be used to reduce fluid overload and relieve pulmonary congestion.  Nitrates: Nitrates such as nitroglycerin may be given to reduce the workload of the heart and improve symptoms.  ACE inhibitors: ACE inhibitors may be started to reduce the risk of further cardiac events and improve outcomes.  Monitoring: The patient should be monitored closely for any signs of worsening heart failure, including changes in vital signs, ECG, and cardiac enzymes. 11. A 45 years old male smoker presented with chest pain. (DU-13Ju, 10Ju) a. Mention the important characteristics of pain which will suggest MI.
  • 21. Professor DR Md . TOUFIQUR RAHMAN , FCPS, MD Professor & Head, Cardiology, CMMC, Manikganj drtoufiq19711@yahoo.com; drtoufiq1971@gmail.com b. Give an outline of management of acute myocardial infraction. a. The important characteristics of chest pain suggestive of myocardial infarction are:  Sudden onset of pain  Chest pain that is severe and crushing in nature  Pain that radiates to the left arm, neck, jaw, back, or shoulder  Pain that is not relieved by rest or nitroglycerin  Pain that lasts for more than 20 minutes b. The management of acute myocardial infarction includes the following steps:  Aspirin: Administer 325mg of aspirin as soon as possible.  Oxygen: Administer oxygen if the patient is hypoxic.  Nitroglycerin: Administer sublingual nitroglycerin for chest pain.  Analgesics: Administer pain relief, such as morphine, if the patient has persistent chest pain.  Reperfusion: Reperfusion therapy should be given within 12 hours of symptom onset. This includes either thrombolytic therapy or percutaneous coronary intervention (PCI).  Anticoagulants: Administer anticoagulant therapy, such as heparin or low molecular weight heparin.  Antiplatelet therapy: Administer antiplatelet therapy, such as clopidogrel or ticagrelor.  Beta blockers: Administer beta blockers in the absence of contraindications.  ACE inhibitors or ARBs: Administer angiotensin-converting enzyme (ACE) inhibitors or angiotensin receptor blockers (ARBs) in the absence of contraindications.  Statins: Administer statins to all patients with acute myocardial infarction. In addition to the above steps, it is important to monitor the patient for complications and provide appropriate management. Close monitoring of vital signs, cardiac rhythm, and oxygen saturation is necessary. Patients with cardiogenic shock may require mechanical support, such as an intra-aortic balloon pump. Patients with arrhythmias may require antiarrhythmic therapy or electrical cardioversion. 12. A 50 years old smoker presents with acute central chest pain. (DU-12Ju) a. What historical features and physical sign will suggest acute coronary syndrome? b. Mention the immediate management measures in this case. a. Historical features and physical signs that may suggest acute coronary syndrome in a 50-year- old smoker with acute central chest pain include:  Sudden onset of pain  Crushing or squeezing pain, often described as "heaviness" or "tightness"  Pain may radiate to the arms (usually left), neck, jaw, back or upper abdomen  Shortness of breath
  • 22. Professor DR Md . TOUFIQUR RAHMAN , FCPS, MD Professor & Head, Cardiology, CMMC, Manikganj drtoufiq19711@yahoo.com; drtoufiq1971@gmail.com  Nausea or vomiting  Sweating  Pallor or grayish color of the skin  Irregular heartbeat  Decreased blood pressure Physical examination may reveal:  Rapid or irregular heartbeat  Low blood pressure  Crackles in the lungs (suggesting heart failure)  Murmurs (suggesting valvular heart disease) b. The immediate management measures for a patient with acute coronary syndrome include:  Oxygen therapy: to maintain oxygen saturation above 94%  Aspirin: to prevent further clotting  Nitroglycerin: to relieve chest pain and improve blood flow  Morphine: if nitroglycerin fails to relieve pain or if the patient is anxious  Beta blockers: to reduce heart rate and blood pressure and prevent further damage to the heart muscle  Anticoagulants: to prevent further clotting and reduce the risk of stroke  Reperfusion therapy: either fibrinolytic therapy or percutaneous coronary intervention (PCI) to restore blood flow to the heart muscle. After the immediate management measures, the patient will need further evaluation, risk stratification and long-term management to prevent further cardiac events. * 13. Give complications of MI. (DU-11Ja, 10Ja) Complications of MI (Myocardial Infarction) can include:  Arrhythmias: Abnormal heart rhythms due to the damage to the heart muscle can lead to various types of arrhythmias, including ventricular tachycardia or fibrillation, which can be life-threatening.  Heart failure: MI can lead to heart failure, which occurs when the heart can't pump enough blood to meet the body's needs. This can lead to symptoms such as shortness of breath, fatigue, and swelling in the legs and ankles.  Cardiogenic shock: This is a severe form of heart failure where the heart is unable to pump enough blood to the vital organs, leading to organ failure and potentially death.  Pericarditis: Inflammation of the lining surrounding the heart can occur following an MI.
  • 23. Professor DR Md . TOUFIQUR RAHMAN , FCPS, MD Professor & Head, Cardiology, CMMC, Manikganj drtoufiq19711@yahoo.com; drtoufiq1971@gmail.com  Thrombosis and embolism: Blood clots can form in the damaged heart muscle and may break off and travel to other parts of the body, causing blockages.  Ventricular aneurysm: MI can lead to a bulge in the weakened heart muscle, which can cause arrhythmias, heart failure, and further MI.  Dressler's syndrome: This is a rare autoimmune response to an MI, which can cause fever, chest pain, and pericarditis.  Stroke: Blood clots formed during MI can break off and travel to the brain, causing a stroke.  Post-infarction angina: Chest pain can persist even after the MI has resolved.  Sudden cardiac death: This can occur due to arrhythmias or other complications of MI. 1. A 30 years old man recently detected as having hypertension. (DU- 20m) (a) Make a check list of clinical history to find out the causes of hypertension. (b) Write down the investigation plan for him. (a) Check list of clinical history to find out the causes of hypertension:  Family history of hypertension  History of previous kidney disease or renal abnormalities  Sleep apnea or other respiratory issues  Hormonal disorders such as thyroid disease or Cushing's syndrome  Alcohol consumption  Smoking history  Drug history, including over-the-counter medications and illegal substances  Dietary habits and physical activity levels  Stressful life events or chronic stress (b) Investigation plan for a patient with hypertension may include:  Blood tests: To check for any underlying health conditions, such as diabetes, kidney disease, or high cholesterol.  Urine tests: To check for any kidney abnormalities or protein in the urine.  ECG (Electrocardiogram): To check for any abnormalities in heart rhythm or structure.
  • 24. Professor DR Md . TOUFIQUR RAHMAN , FCPS, MD Professor & Head, Cardiology, CMMC, Manikganj drtoufiq19711@yahoo.com; drtoufiq1971@gmail.com  Ambulatory blood pressure monitoring (ABPM): A 24-hour blood pressure measurement to assess blood pressure variability and determine if the patient has white-coat hypertension or masked hypertension.  Echocardiogram: To evaluate the structure and function of the heart and to rule out any underlying heart disease.  Imaging tests: Such as CT scan, MRI, or angiography to evaluate blood vessels and to check for any abnormalities that may be contributing to hypertension.  Stress test: To evaluate the patient's heart function during exercise and to determine the presence of any underlying heart disease.  Genetic testing: In some cases, genetic testing may be performed to determine if the patient has a genetic predisposition to hypertension or underlying heart disease. The investigation plan may vary based on the individual patient's history, symptoms, and physical examination findings. 2. A 35 year old male recently detected as hypertension for 3 months. (DU-19Ja) a. Make a check list and physical sign to find out the causes of hypertension. b. Write down the investigation plan for him. a. The checklist and physical signs to find out the causes of hypertension in a 35-year-old male are: Checklist:  Family history of hypertension  History of kidney disease  History of diabetes mellitus  Lifestyle factors: diet, exercise, tobacco and alcohol use  Medication history  Endocrine disorders such as Cushing's syndrome or hyperthyroidism  Sleep apnea Physical signs:  Elevated blood pressure reading on at least two occasions  Evidence of end-organ damage such as retinal changes, proteinuria, or left ventricular hypertrophy  Obesity  Thyroid enlargement or nodules  Abdominal bruit  Palpable renal masses
  • 25. Professor DR Md . TOUFIQUR RAHMAN , FCPS, MD Professor & Head, Cardiology, CMMC, Manikganj drtoufiq19711@yahoo.com; drtoufiq1971@gmail.com b. The investigation plan for a 35-year-old male with hypertension may include:  Complete blood count (CBC)  Fasting blood glucose level  Lipid profile  Renal function tests including serum creatinine, blood urea nitrogen (BUN), and estimated glomerular filtration rate (eGFR)  Urinalysis  Electrocardiogram (ECG)  Echocardiogram  Chest X-ray  Thyroid function tests  Sleep study (if sleep apnea is suspected)  24-hour urine collection for catecholamines and metanephrines (if pheochromocytoma is suspected) Additional tests may be ordered based on the initial findings and clinical suspicion. The investigation plan should also include regular monitoring of blood pressure and close follow-up with a healthcare provider to assess response to treatment and adjust management accordingly.